You are on page 1of 110

AP® MICROECONOMICS

1999 SCORING GUIDELINES


Question 1
Correct Answer
Part (a) With fewer firms producing textiles, the supply of textiles is reduced; the
supply curve shifts to the left (inward). With a higher price for textiles, the
quantity supplied increases along an unchanged supply curve.
Part (b) The tariff raises the per unit supply price of all units of imported textiles;
the supply curve in market B shifts to the left (inward), leading to a higher
equilibrium price and a lower equilibrium quantity of imported textiles. Since
imported and domestically produced textiles are close substitutes, the increase in
the price of imported textiles leads to an increase in the demand for domestically
produced textiles (market A). Both the equilibrium price and quantity increase in
market A.
Part (c) A higher wage for textile workers leads to a reduced supply of
domestically produced textiles. In market A, the supply curve shifts to the left
(inward), and there is an increase in the equilibrium price and a reduction in the
equilibrium quantity.
Part (d) The individual firm is a wage taker in the hiring of labor. Thus the firm
faces a perfectly elastic (horizontal) labor supply, and the firm is able to hire all
the labor it wishes at this wage rate. Given that the wage rate has increased, the
firm's labor supply, while remaining perfectly elastic, shifts up to reflect the
higher wage. The firm's labor demand, its marginal revenue product of labor
curve, shifts out (increases) because the price of textiles increased (in part c).
[While not required in the answer, the intersection of the firm's new labor supply
and new labor demand should show a reduction in the quantity of labor employed
by the firm. Given that all firms are collectively producing fewer textiles (part c),
the representative firm will produce fewer textiles and employ fewer units of
labor.]

Scoring Rubric
Part (a) = 2 points, Part (b) = 4 points, Part (c) =1 point, Part (d) = 2 points; 9
Points in Total
Part (a)
i. inward shift in supply; a decrease in supply (1 point)
ii. movement along the supply curve; an increase in the quantity
supplied or supply curve does not change (1 point)

Copyright 2001 by College Entrance Examination Board. All rights reserved.


Advanced Placement Program and AP are registered trademarks of the College Entrance Examination Board.

2
AP® MICROECONOMICS
1999 SCORING GUIDELINES
Question 1 (cont.)
Part (b)
i. Import Market B:
shift inward (decrease) supply: need correct graph (1 point) leading
to increased price and reduced quantity in market B (either in a
correct graph or a verbal explanation) (1 point)
ii. Domestic Market A:
shift out (increase) in demand: need correct graph + verbal linkage
to the price increase in Import Market B in part (b-i) above (1
point) leading to increased price and increased quantity in market
A (either in a correct graph or a verbal explanation) (1 point)
Part (c)
Supply shifts in (decreases): leading to a price increase and a decrease in
quantity on a correctly labeled graph (1 point)
Part (d)

• Firm's labor hiring situation: NOT the market

• MRPL shifts out due to an increase in the output price (1 point)

• Labor supply to the firm (must be perfectly elastic or horizontal)


shifts up following the increase in the wage rate (1 point)
[Notes: 1. The firm's quantity of labor hired should fall since each firm is
producing less output from Part (c). However, the two points may be
awarded even if that conclusion (which is not asked for in the question) is
not reached. 2. MRP may be labeled DL if the axes are properly labeled
(wage on the vertical axis and employment, or quantity of labor, on the
horizontal axis); the graph must clearly related to the firm's hiring of labor
and not that of the labor market.]
Note: Besides counting points, the answer may be looked at as a whole and
ultimately judged by its overall quality. The final total should mean something in
terms of the overall quality of the answer. An 8 or 9 should reflect an excellent
answer (a 9 is not necessarily a perfect answer); a 6 or 7, a good answer; a 4 or 5,
an adequate answer; a 3 a seriously deficient answer, but still an answer; a 2 an
answer signifying nothing except one sustained argument; and a 1, containing
only a correct, relevant-to-the question statement. A 0 has no relevant economic
answer to the question. A dash (-) is given for an unresponsive or blank answer.

Copyright 2001 by College Entrance Examination Board. All rights reserved.


Advanced Placement Program and AP are registered trademarks of the College Entrance Examination Board.

3
AP® MICROECONOMICS
1999 SCORING GUIDELINES
Question 1 (cont.)

Purpose of the Question and Commentary on Students' Responses


This long microeconomic question tests students' understanding of product
markets and the firm's labor hiring decision. The first part of the question tests an
understanding of the difference between a change in supply and a change in the
quantity supplied. Many students failed to demonstrate their understanding of this
fundamental difference. In part (b), students are asked to assess the impact of a
tariff on both the market for imported textiles and for domestically produced
textiles. Part (c) asks for the impact of a wage increase for labor producing
textiles within the United States. In part (d), the wage increase from part (c) and
the resulting increase in the price of domestically produced textiles are needed to
analyze the individual firm's hiring of textile workers. Very few students made the
transition from the market analysis — Parts (a),(b), and (c) — to the firm analysis,
Part (d). Frequently students provided a graph and explanation for the labor
market, but not for the individual firm. Even fewer students realized that the firm
was a wage taker that faced a supply of labor that was perfectly elastic at the wage
rate. In this question the supply of labor to the firm, — while remaining perfectly
elastic — shifts up, reflecting the higher wage rate. Given that the price of textiles
has increased, the marginal revenue product of labor (the firm's labor demand
curve) shifts out. [Note: the individual firm, producing less output, will employ
less labor; the intersection of the new labor supply and the new labor demand
should result in fewer workers hired.] Students, year after year, seem to have
major difficulty with questions that use the marginal revenue product of labor
concept or that relate to the hiring of inputs.

Copyright 2001 by College Entrance Examination Board. All rights reserved.


Advanced Placement Program and AP are registered trademarks of the College Entrance Examination Board.

4
AP® MICROECONOMICS
1999 SCORING GUIDELINES
Question 2
Correct Answer
Part (a) With equal amounts of resources and identical technologies, each country
can produce the same maximum quantity of wheat. Thus, neither country has an
absolute advantage in wheat production. County B has the absolute advantage in
cloth production because it can produce more than country A.
Part (b) For Country A the opportunity cost of a bushel of wheat is 1 yard of
cloth; for Country B, the opportunity cost of a bushel of wheat is 3 yards of cloth.
Since Country A has the lower opportunity cost for producing wheat, Country A
has comparative advantage in wheat production. Alternatively, Country B has the
comparative advantage in the production of cloth.
Part (c) Given that comparative advantage is the basis for trade, Country B will
specialize in the production of cloth, exporting cloth and importing wheat.
Part (d) County B will import wheat, exchanging 2 units of cloth for one unit of
wheat. The country benefits from trade because if it produced wheat itself, a unit
of wheat would be more expensive, costing 3 units of cloth.

Scoring Rubric
Part (a) = 1 Point, Part (b) = 2 Points, Part (c) = 1 Point, Part (d) = 1 Point; 5
Points in Total
Part (a)
Wheat: neither country has an absolute advantage, identical outputs (with
the same resources)
Cloth: Country B, as it produces more output (with the same resources) (1
point)
Part (b)
Country A has comparative advantage in wheat; Country B in cloth (1
point)
Show opportunity costs: Country A gives up 1 cloth for 1 wheat; Country
B gives up 3 cloth for 1 wheat; relatively more expensive for Country B to
produce wheat (1 point)

Copyright 2001 by College Entrance Examination Board. All rights reserved.


Advanced Placement Program and AP are registered trademarks of the College Entrance Examination Board.

5
AP® MICROECONOMICS
1999 SCORING GUIDELINES
Question 2 (cont.)
Part (c)
Country B will import wheat because it has comparative advantage in
cloth (and will specialize in cloth production, getting its wheat more
cheaply via trade) Key point is comparative advantage (1 point)
[The student may also state that in Country B the opportunity cost of
wheat is higher than in Country A; thus, Country B will import wheat.]
Part (d)
For country B, via trade a unit of wheat only costs 2 units of cloth, while
by domestic production a unit of wheat would cost 3 units of cloth. Also
the student may explain that trade increases B's consumption possibilities
by making it possible to consume more of both goods (1 point)
Note: Besides counting points, the answer may be looked at as a whole and
ultimately judged by its overall quality. The final total should mean something in
terms of the overall quality of the answer. A 5 should reflect an excellent answer,
but not necessarily a perfect one; a 4, an excellent answer with a flaw; a 3, a good
answer; a 2, an adequate answer; a 1, a seriously deficient answer, but still an
answer. A 0 has no relevant economic answer to the question. A dash (-) is given
for an unresponsive or blank answer.

Purpose of the Question and Commentary on Stidents' Responses


This question relates to absolute advantage, comparative advantage, opportunity
cost, and the gains from trade. Students tended to have an understanding of the
concepts of absolute and comparative advantage. Often, however, explanations
concerning opportunity cost were lacking. Frequently students did not clearly
state that the opportunity cost of producing a unit of wheat in Country A (1 unit of
cloth) was less than the opportunity cost of producing a unit of wheat in Country
B (3 units of cloth); and that this differential resulted in A having comparative
advantage in wheat and B having a comparative advantage in cloth. Students also
did not demonstrate a good understanding of the gains from trade: either acquiring
a product at lower cost through trade than through domestic production or
enjoying increased consumption of both goods (consuming beyond the production
possibilities curve).

Copyright 2001 by College Entrance Examination Board. All rights reserved.


Advanced Placement Program and AP are registered trademarks of the College Entrance Examination Board.

6
AP® MICROECONOMICS
1999 SCORING GUIDELINES
Question 3
Correct Answer
Part (a) The competitive firm is a price taker and faces a horizontal demand curve
at the market price. The marginal revenue curve is also horizontal at the market
price, coinciding with the demand curve. The monopoly firm faces the downward
sloping market (or industry) demand curve. The marginal revenue curve is below
the demand curve. [For a linear demand curve, the slope of the marginal revenue
curve is twice that of the demand curve.]
Part (b) For the competitive firm, marginal revenue equals demand. The
competitive firm sells each unit of output at a constant price. Thus, the additional
revenue associated with an additional unit of output (or marginal revenue) is equal
to the market price.
Part (c) For the monopoly firm, marginal revenue is less than price. Unlike the
perfectly competitive firm, the monopolist is a price setter. For the monopoly firm
to sell an additional unit of output, the firm must lower price on all units of
output. Thus, the extra revenue received from selling an additional unit of output
is offset by the selling of other units at a lower price.

Scoring Rubric
Part (a) = 2 points, Part (b) = 1 point, Part (c) = 2 points; 5 Points in Total
Part (a)
Correct graph showing the relationship between demand and marginal
revenue for the competitive firm. (1 point)
Correct graph showing the relationship between demand and marginal
revenue for the monopoly firm. (1 point)
Part (b)
D=MR:the firm is a price taker and can sell all output at the market price
(1 point)

Copyright 2001 by College Entrance Examination Board. All rights reserved.


Advanced Placement Program and AP are registered trademarks of the College Entrance Examination Board.

7
AP® MICROECONOMICS
1999 SCORING GUIDELINES
Question 3 (cont.)
Part (c)
D (or P) > MR
Explaining that the monopoly has control over price (1 Point)
Indicating that to sell additional units, price must be lowered on all units
(1 Point)
Note: Besides counting points, the answer may be looked at as a whole and
ultimately judged by its overall quality. The final total should mean something in
terms of the overall quality of the answer. A 5 should reflect an excellent answer,
but not necessarily a perfect one; a 4, an excellent answer with a flaw; a 3, a good
answer; a 2, an adequate answer; a 1, a seriously deficient answer, but still an
answer. A 0 has no relevant economic answer to the question. A dash (-) is given
for an unresponsive or blank answer.

Purpose of the Question and Commentary on Students' Responses


Using both a competitive firm and a monopoly firm, this question tests students'
understanding of demand and marginal revenue. Many students failed to see that
the competitive firm is a price taker and has a perfectly elastic (horizontal)
demand for its output. As a result, the firm's marginal revenue is also perfectly
elastic, is equal to price, and coincides with the firm's demand curve. All AP
Microeconomics courses should clearly explain the differences between a firm
that is a price taker (competitive firm) and a price setter (monopolist).

Copyright 2001 by College Entrance Examination Board. All rights reserved.


Advanced Placement Program and AP are registered trademarks of the College Entrance Examination Board.

8
2000

Scoring Guideline for Microeconomics Question 1:

[2+2+1+2+1+1+2=11]

Part (a)
(i) MR = MC for output (1 point)
(ii) P from Demand curve (1 point)

Part (b)
(i) Q1 and P5 (or Point B) (1 point)
ii) A to D (or range for which MR > 0) (1 point)
• It is not necessary for them to acknowledge that D technically shouldn’t be included
in this line segment.
• While other line segments (AB, AC, BC, BD, CD) are elastic, they include only a
portion of the line segment that is elastic and should not be given credit.
• Often, the answer to A) will be found in B), and can be acceptable as a correct
answer to A).
• Also, the student may attempt to answer this (and other) part of the
question with a simplified graph of his/her/its own, such as the one
below. If it is correctly labeled and is correct, this is acceptable.
P

MC Profit maximizing output here is Q1


P1 A Profit maximizing price is P1
A is the profit-maximizing point
Demand

Output
Q1 MR

Part (c)
Q2 and P4, (or point C) (1 point).

Part (d)
(i) ABP5 (monopoly) (1 point)
• If the answer to d) I) is incorrect but consistent with the student’s answer in b), credit
should be given.
(ii) ACP4 (perfect competition) (1 point)
• Again, there is a consistency problem; if the answer to ii) is incorrect but is consistent
with the (incorrect) answer the student gave in c), credit should be given.

Part (e)
P = MC ,or
MB = MC (1 point)
• This definition is entirely self-contained; there need be no reference to the graph to
get full credit.
2000

Part (f)
Q2 ( 1point)
• If this answer is incorrect the student receives no credit even if it is consistent with
part e)

Part (g)
A per-unit subsidy. Merely the word “subsidy” is sufficient. (1 point).
ii) An explanation that the subsidy either (1 point)
1) lowers MC, or
2) raises MR, or
3) shifts the demand curve out.
• A statement that output increases is not sufficient.

Scoring Guideline for Microeconomics Question 2:

Question 2 [1+1+2+1+1 = 6 Points]


Part (a)
(1 point) Perfect competition (or pure competition) since price of output is given to the
firm
• “Because the firm is a price-taker” is also acceptable.

Part (b)
(1 point) Perfect competition since the wage rate is given to the firm (1 point)
• “Because the firm is a wage-taker” is also acceptable.

Part (c)
(2 points) 1 point for 5 workers; 1 point for the explanation

The MRP > w for 5 workers but not for 6.


• It is not sufficient to point out that the firm earns a profit (or its greatest profit) at 5
workers; some reference must be made to the fact that MRC (MFC, MC of labor, or
wages) is closest to MRP and is still greater than MRP at that level of output.

• The student must use MRP in the answer; MR is not sufficient.

Part (d)
(1 point) for 46 units of output. The output level must be consistent with the units of
labor selected in part c. In this case, a student who has the wrong number of labor in part
c. may still earn the point here if the output level is consistent with that quantity of labor
indicated by the student in part c.
2000

Part (e)
TR: $138 (46 x $3) (1 point)
- TVC 55 (5 x $11)
- TFC 10
= Economic Profit $ 73

• Must be consistent with the student’s answer above.


• They do not have to show the actual profit; if they state that a profit is being earned
because TR > TC they receive credit.

Scoring Guidelines for Microeconomics Question 3

[2+1+2 = 5 Points]

Part (a) Market for Imported Shoes


(i) Correct graph (Labels: D,S,P, & Q) that includes an inward shift in supply (1
point)
(ii) P and Q changes correctly explained using the student’s graph of a) i). (1
point)

Part (b) Market for Domestic Shoes


Correct graph that includes a demand shift consistent with the price change in Part a
with
both P and Q consistent with the graph drawn (1 point)

Part (c)
(i) With an elastic demand for imported shoes, the increased price of imported
shoes will lead to a reduction in total expenditures ( 1 point).
• A student who has an incorrect answer in part a) may earn this point as long as the
analysis is consistent with the answer in part a).

(ii) Discussion of the proportional change in P and Q to explain the expenditure


change. (1 point)

• Be careful to work with the student’s output change in Part a.


AP® MICROECONOMICS
2001 SCORING GUIDELINES

Question 1
Correct answer:
The firm has a perfectly elastic (or horizontal) marginal revenue curve that is equal to the market price.
The firm produces the output level where marginal revenue equals marginal cost. The economic profit of
the firm is the area bounded by the quantity produced multiplied by the difference between price and
average total cost (P-ATC) at that output level.
With economic profits, new firms will enter the industry. The market supply will shift outward with the
entry of firms, and market price will fall. The process continues until a long-run equilibrium is
established. At this equilibrium, the market price is equal to the minimum of the long-run average cost of
the typical firm. Each firm produces where P=MR=MC, which is the level of output that corresponds to
the minimum of the long-run average cost. The firm makes zero economic profits.
A price control below the long-run equilibrium price but above the firm’s average variable cost will result
in short-run production. Since the price has fallen, the firm’s marginal revenue falls. The firm’s output
level, where MR=MC, will also decrease. Since the firm is producing less output, total cost falls. Since
both the firm’s price and quantity have fallen, total revenue falls.

3+5+4 = 12

(a) 3 points

A correctly labeled graph of the firm’s situation is sufficient for full credit on part (a). An
explanation is not necessary. The breakdown is as follows:

(i) 1 point A horizontal MR shown.


(ii) 1 point Show (and indicate) that output should be where MR = MC.

Copyright © 2001 by College Entrance Examination Board. All rights reserved.


Advanced Placement Program and AP are registered trademarks of the College Entrance Examination Board.

2
AP® MICROECONOMICS
2001 SCORING GUIDELINES

Question 1 (cont.)
(iii) 1 point Show the area of profit as the rectangle whose width is the distance between 0
and the equilibrium quantity and whose height is the distance between the P(AR) and ATC at
that quantity.

(b) (5 points in total)

(i) 3 points The breakdown is as follows:


1 point showing correctly drawn side-by-side graphs of the firm and the market,
with both graphs correctly labeled.
1 point showing a market supply curve shifting out and a market price decreasing,
with this price being used in the firm graph.
1 point explaining that with economic profits, more firms enter the market.
(ii) 2 points The breakdown is as follows:
1 point showing the industry equilibrium price and quantity. Students can either
use a vertical line from the x axis to the equilibrium point, and one from the
equilibrium point to the y axis (and label each axis) or else they can mark the
equilibrium point with a letter and somehow indicate that the point represents the
equilibrium.
1 point showing the long-run equilibrium price and quantity for the firm at the
minimum ATC, indicating it via either of the methods discussed above in b (ii).
(c) 4 points A graph is not necessary to answer this question. The point breakdown
is as follows:
(i) 1 point For indicating that marginal revenue has fallen because P has been
lowered.
(ii) 1 point For indicating that output has fallen because MR = MC is at a lower q.
(iii)1 point For indicating that total costs fell because output fell.

Copyright © 2001 by College Entrance Examination Board. All rights reserved.


Advanced Placement Program and AP are registered trademarks of the College Entrance Examination Board.

3
AP® MICROECONOMICS
2001 SCORING GUIDELINES

Question 1 (cont.)

(iv) 1 point For indicating that price and quantity fell. It is not sufficient to state that
merely one of the two fell: either both are explicitly stated, or else the student clearly
shows on the graph that the new equilibrium has a lower P and Q and that because of
this, total revenue falls.

Copyright © 2001 by College Entrance Examination Board. All rights reserved.


Advanced Placement Program and AP are registered trademarks of the College Entrance Examination Board.

4
AP® MICROECONOMICS
2001 SCORING GUIDELINES

Question 2
Correct Answer:
The student should show a market supply curve that includes only private costs of production. For a given
market demand curve, there will be an equilibrium price and quantity of output. There should be a second
supply curve that incorporates all costs of production, including the external costs. The socially optimum
level of output is found at the intersection of the market demand and the supply curve that incorporates all
costs, both private and external. With the same market demand curve, at the social optimum, the
equilibrium price should be higher and the equilibrium quantity lower. In essence, the unregulated private
market will produce too much output at too low a unit price.
To achieve the socially optimum level of output the government could introduce a unit tax on output. If
properly chosen, this tax could raise the supply curve with only private costs to intersect at the socially
optimum output level. Alternatively, quantity controls or pollution permits could be used to correct the
overproduction.

3 + 1 = 4 points

(a)
(i) 1point A simple supply and demand diagram, correctly labeled, is sufficient for full credit.
(ii) 1 point Recognizing that the negative externality results in a difference between the
private supply curve and the socially optimum supply curve.
• The second supply curve must be labeled “socially optimal” or MSC or something to that
effect.
• They need not explicitly identify the externality.
• Simply labeling curves S1 and S2 w/o an explanation is insufficient.
• A firm graph is incorrect.
The difference in two supply curves must be due to the existence of external costs:
• MSC > MPC
• Cost not accounted for in the private market
• Swith external costs > Sprivate
They can not get credit for shifting supply due to a tax — it confuses causality.

Copyright © 2001 by College Entrance Examination Board. All rights reserved.


Advanced Placement Program and AP are registered trademarks of the College Entrance Examination Board.

5
AP® MICROECONOMICS
2001 SCORING GUIDELINES

Question 2 (cont.)
(iii) 1 point Showing a higher price and lower quantity in the socially optimum equilibrium.
(b) 1 point Various answers will suffice:
• A unit tax
• Quantity restriction
• Effective price ceiling (not a price floor) that leads to production of Q1
• Pollution permits

Copyright © 2001 by College Entrance Examination Board. All rights reserved.


Advanced Placement Program and AP are registered trademarks of the College Entrance Examination Board.

6
AP® MICROECONOMICS
2001 SCORING GUIDELINES

Question 3
Correct Answer:
Worker number 3 has the highest marginal product (i.e., 60 – 35 = 25 cars washed). With additional
workers the marginal product falls. This is consistent with the Law of Diminishing Returns. That law
states that as more units of a variable input (labor) are employed with a fixed input, output will eventually
increase at a decreasing rate. The sixth worker would never be hired since the marginal product of that
worker is negative (80-85= -5 cars). A firm would never hire a unit of an input that reduces total output.
The firm would be willing to pay the fourth worker as much as its marginal revenue product or $90 per
day found by multiplying the price of a car wash by the number of cars washed by the fourth worker (i.e.,
$6 x 15 = $90).

1 + 2 + 1 + 1 = 5 points in total

(a) 1 point 3rd worker


(b) 1 point Diminishing marginal returns
1 point Definition of diminishing marginal returns that includes both variable and fixed inputs

(c) 1 point Negative marginal product for the 6th worker


Also accepted: negative returns, output falls, or MRP < 0.

(d) 1 point $90 (P x MP or $6 x 15 = $90)

Copyright © 2001 by College Entrance Examination Board. All rights reserved.


Advanced Placement Program and AP are registered trademarks of the College Entrance Examination Board.

7
AP® MICROECONOMICS
2002 SCORING GUIDELINES

Question 1

Correct Answer:

Part a: The student should recognize that Claire is a monopolist. The firm (Claire) would have a downward-
sloping product demand curve with a marginal revenue curve below the demand curve. The profit-maximizing
level of output would be where marginal revenue equals marginal cost. The product price would be found on the
demand curve, above the profit maximizing output level. The firm’s economic profit would be the rectangle
bordered vertically by the distance between price (P) and average total cost (ATC) and horizontally by the output
level (Q).

Part b: The student should show a competitive labor market with a downward-sloping labor demand curve and
an upward sloping labor supply curve. There will be an equilibrium wage per unit of labor and equilibrium
quantity of labor. Claire, as a wage taker, will face a perfectly elastic labor supply at the equilibrium market
wage. The number of workers hired by Claire is found at the intersection of Claire’s downward-sloping marginal
revenue product of labor function (labor demand) and the perfectly elastic labor supply (at the market wage).

Part c: Product X is now sold in a perfectly competitive product market. The student should show a competitive
output market with an equilibrium price and quantity. Claire and the other competitive firms have an output
demand that is now perfectly elastic at the market-determined price of output. In the long-run equilibrium each
firm will produce where the output price (also, the firm’s marginal revenue) is equal to marginal cost at minimum
average total cost.

Grading Rubric:

4+3+3 = 10 points for parts a, b, c

(a)

Profit-maximizing output level and price, profits - 4 points


1 Point: correctly labeled graph with downward-sloping D and MR, with D>MR
1 Point: Q at MR = MC
1 Point: P from D, above the MR =MC point
1 Point: Profit rectangle properly shown: must use (P – ATC) x Q

Copyright © 2002 by College Entrance Examination Board. All rights reserved.


Advanced Placement Program and AP are registered trademarks of the College Entrance Examination Board.

2
AP® MICROECONOMICS
2002 SCORING GUIDELINES

Question 1 (cont’d.)

(b)

Labor hiring and wage rate – 3 points


1 Point: correctly labeled labor market graph: must have an upward-sloping labor supply and downward-
sloping labor demand
1 Point: correctly labeled firm graph as a part of the side-by-side graphs: must show linkage from labor
market to indicate firm is a wage-taker (perfectly elastic labor supply curve) at the market wage
1 Point: for the correct number of workers for Claire: where Wage (labor supply, SL) = MRP (downward
sloping labor demand curve, labeled MRP or DL)

(c)

Product Market-perfect competition – 3 points


1 Point: correctly labeled graph of the market: showing equilibrium price and quantity of output
1 Point: correctly labeled graph of the firm as a part of the side-by-side graphs: must show linkage from
the product market to indicate that the firm is a price-taker (perfectly elastic demand curve at the
market output price)
1 Point: showing the firm’s output level where MR=MC at minimum ATC

Copyright © 2002 by College Entrance Examination Board. All rights reserved.


Advanced Placement Program and AP are registered trademarks of the College Entrance Examination Board.

3
AP® MICROECONOMICS
2002 SCORING GUIDELINES

Question 1 (cont’d.)

Commentary:

This long microeconomics question tested the student’s understanding of both output and input markets, as well
as the linkage between the market and the individual firm. Also, the question included two different market
structures, monopoly and perfect competition. The question was quite effective in separating across different
grading points. As a general observation, it should be noted that too many students seemed unaware of the
meaning of side-by-side graphs, needed in parts b and c. To show convincingly the links between a market and an
individual competitive firm, these graphs are necessary.

Too frequently the student’s labor market graph could not be distinguished from the output market graph; the
student would have Q on the vertical axis and P on the horizontal axis for both graphs.

In both parts b and c, students frequently did not separate the market from the individual firm. A critical concept
in perfect competition, both in the labor market and in the output market is that of “price taking.” For the firm
hiring labor in a perfectly competitive labor market, the price of labor becomes the firm’s perfectly elastic labor
supply, with the individual firm able to hire all the labor it wishes at the market-determined wage. Similarly, in
the output market, the individual firm faces a perfectly elastic product demand at the market-determined output
price.

Copyright © 2002 by College Entrance Examination Board. All rights reserved.


Advanced Placement Program and AP are registered trademarks of the College Entrance Examination Board.

4
AP® MICROECONOMICS
2002 SCORING GUIDELINES

Question 2

Correct Answer:

Part a: Within the chemical industry (or market) at the unregulated level of output the marginal social cost of
production exceeds the marginal social benefit. In other words, with this negative externality, there is an over
allocation of resources to the chemical industry; the level of output is greater than the efficient level. The
government should introduce a per unit tax on output, raising the marginal private cost of production and
reducing output. Alternatively, the government could introduce some measure to reduce directly the level of
output.
Part b: National defense is a public good. Individuals have an incentive to withhold their true demand or
willingness to pay for the good, i.e., the free-rider problem. Thus, at the level of output produced the marginal
benefit of national defense exceeds the marginal cost of national defense; there is an under allocation of
resources to national defense. The government could assume production of national defense and tax all members
of the society to pay for the national defense. Or, a per-unit subsidy to private producers would lead to an
increase in the output of national defense.

Grading Rubric:

6 Points = 3 in part a + 3 in part b

(a) Chemical industry and pollution (3 points)


(i) Acceptable answers include: (1 point)
Too much output
Over allocation of resources to the market
Showing higher than efficient output on graph

Acceptable answers include: (1 point)


MSC > MSB at the unregulated output
MSC > MPC
MC > MB with term “negative externality”

(ii) Acceptable answers include: (1 point)


Tax on output
Quantity restriction
Permits
Liability and lawsuit

Copyright © 2002 by College Entrance Examination Board. All rights reserved.


Advanced Placement Program and AP are registered trademarks of the College Entrance Examination Board.

5
AP® MICROECONOMICS
2002 SCORING GUIDELINES

Question 2 (cont’d.)

(b) National Defense (3 points):


(i) Acceptable answers include: (1 point)
Too little produced
Under allocation of resources
Showing lower than efficient output on graph

Acceptable answers include: (1 point)


MSB > MSC at the unregulated output
MSB > MPB
Free-rider problem

(ii) Acceptable answers include: (1 point)


Public production of national defense
Tax to finance public production of national defense
Subsidy, if there are private producers of national defense

Commentary:
We began this question reminding students that an efficient allocation of resources occurs when the marginal
social cost equals the marginal social benefit. Students then had to assess two situations in which an efficient
allocation of resources does not occur and to explain why inefficiency exists.

Copyright © 2002 by College Entrance Examination Board. All rights reserved.


Advanced Placement Program and AP are registered trademarks of the College Entrance Examination Board.

6
AP® MICROECONOMICS
2002 SCORING GUIDELINES

Question 3

Correct Answer:

Part a: The utility-maximizing consumer will exhaust her income, purchasing quantities of each good such that
for each commodity the marginal utility of the last unit purchased divided by the price of the commodity is equal.
This consumer will purchase 3 apples and 2 oranges. The marginal utility per dollar of each commodity is equal:
10/$1 for apples and 20/$2 for oranges.

Part b: With the increase in income, the consumer will now purchase 4 apples and 4 oranges and have 125 utils
(50 from apples and 75 from oranges).

Part c: With the increase in the price of oranges, the consumer will now purchase 4 apples and 2 oranges and
have 100 utils (50 from apples and 50 from oranges).

Grading Rubric:

Part a, b, and c each worth 2 points for 6 points in total

(a) 3 apples and 2 oranges (1 point)


Marginal analysis: equalization of MU/$ or 10/1 (apples) = 20/2 (oranges) (1 point)
Note: The student may not simply use the maximizing of total utility for the explanation.

(b) 4 apples and 4 oranges (1 point)


50+75 = 125 utils (1point)

(c) 4 apples and 2 oranges (1 point)


50+50 = 100 utils (1 point)

Note: For parts b and c, the reader must work with the student’s apple/orange combination and award a point if
the total utility is consistent with that combination.

Commentary:

Students were able to receive 2 of 6 points for calculating the correct amount of total utility from two incorrect
apple and orange combinations. Far too few students were able to apply the utility-maximizing rule of equalizing
the marginal utility per dollar for each commodity.

Copyright © 2002 by College Entrance Examination Board. All rights reserved.


Advanced Placement Program and AP are registered trademarks of the College Entrance Examination Board.

7
AP® MICROECONOMICS
2002 SCORING GUIDELINES (Form B)

Question 1

Correct Answer:

(a) See the graphs above. The long-run equilibrium price and quantity are labeled Pm and Qm , for the
monopolistically competitive firm, and Pc and Qc for the perfectly competitive firm.

(b) The perfectly competitive firm has a lower price and a larger quantity of output than the monopolistically
competitive firm.

(c) Each of these firms will earn zero economic profits in the long run. With no barriers to entry, the existence of
positive economic profits or economic losses motivates the entry or exit of firms in and out of the industry,
forcing prices to the level of average costs.

(d) Demand is perfectly elastic for the perfectly competitive firm because price is constant, making the
percentage change in price zero for any change in quantity. For the monopolistically competitive firm,
demand is elastic, because MR is positive at Qc.

Copyright © 2002 by College Entrance Examination Board. All rights reserved.


Advanced Placement Program and AP are registered trademarks of the College Entrance Examination Board.

2
AP® MICROECONOMICS
2002 SCORING GUIDELINES (Form B)

Question 1 (cont’d.)

Grading Rubric:

Question 1: 15 points (6 + 2 + 3 + 4)

(a) 6 points (1 point for each graph showing the appropriate cost curves, 1 point each for identifying profit
maximizing quantity at MC=MR, and 1 point each for showing price for each firm–read off the correct demand
curve)

1 point each: two graphs with appropriate cost curves


1 point each: Q indicated for each firm where MR=MC
1 point each: P for each firm read off the correct D curve at correct Q

(b) 2 points (1 point for the price comparison and 1 point for quantity comparison)

1 point: P in perfect competition is lower than P in monopolistic competition.


1 point: Q in PC is smaller than Q in MC

(c) 3 points (1 point each for indicating zero economic profit for each firm and 1 point for explanation)

1 point : firm in perfect competition earns zero economic profit


1 point : firm in monopolistic competition earns zero economic profit
1 point : entry of new firms increases industry output, individual firm’s output decreases, prices will
fall to level of ATC (Correct explanation consistent with the exit of firms is also accepted)

(d) 4 points (1 point each for correctly identifying elasticity for each firm, 1 point each for correct
explanation)

1 point : for the perfectly competitive firm, demand is perfectly elastic


1 point : because P is constant, the percentage change in P is zero
1 point : for the monopolistically competitive firm demand is elastic
1 point : MR is positive in the elastic portion of the demand curve

Copyright © 2002 by College Entrance Examination Board. All rights reserved.


Advanced Placement Program and AP are registered trademarks of the College Entrance Examination Board.

3
AP® MICROECONOMICS
2002 SCORING GUIDELINES (Form B)

Question 2

Correct Answer:

(a) The government could impose a per-unit tax on pollution.

(b) (i) Marginal cost increases because the tax increases the cost per unit of output.

(ii) Perfectly competitive firms select the output level at which MC = P. Since MC has increased, it will
intersect the price line at a lower quantity.

(iii) The supply curve will shift up by the amount of the tax, causing the price to rise.

(c) Efficiency will increase because the tax forces the firms to internalize the full cost of their behavior, making
the marginal private cost equal to the marginal social cost. At the new equilibrium, the marginal social cost
will equal the marginal social benefit, indicating a socially efficient outcome.

Grading Rubric:

Question 2: 8 points ( 1 + 5 + 2)

(a) 1 point for a correct policy

1 point: government can impose a per-unit tax on pollution, sell pollution permits,
restrict quantity, or require emissions reduction equipment

(b) 5 points (1 point for MC, 1 for direction of output and 1 for explanation, 1 for
direction of price and 1 for explanation)

1 point: MC increases, since the tax increases the cost per unit of output
1 point: output will decrease
1 point: explanation that MC has shifted up and the firm will select new output
level where the new MC equals P
1 point: price will increase
1 point: explanation that the supply curve will shift up by the amount of the tax, causing the price to rise

(c) 2 points (1 point for efficiency and 1 for explanation)

1 point: efficiency will increase


1 point: explanation that the tax shifts the supply curve up to account for
the social cost of pollution, and at the new equilibrium the MSC equals the MSB

Copyright © 2002 by College Entrance Examination Board. All rights reserved.


Advanced Placement Program and AP are registered trademarks of the College Entrance Examination Board.

4
AP® MICROECONOMICS
2002 SCORING GUIDELINES (Form B)

Question 3

Correct Answer:

(a) (i) See the labor market graph above. The demand for labor decreases and the demand curve shifts to the left.
With the decrease in demand for labor in Bazra, the equilibrium wage rate falls from W to W' and the
number of employed workers falls from L to L'.

(ii) At the new wage rate W', the new quantity of labor demanded equals the quantity of labor supplied, so
the number of workers looking for work who cannot find it is zero.

Copyright © 2002 by College Entrance Examination Board. All rights reserved.


Advanced Placement Program and AP are registered trademarks of the College Entrance Examination Board.

5
AP® MICROECONOMICS
2002 SCORING GUIDELINES (Form B)

Question 3 (cont’d.)

(b) (i) As indicated in the graph below, an effective minimum wage will increase the wage rate and decrease the
quantity of workers employed in Bazra.

(ii) The number of workers looking for work who cannot find it increases from zero to the positive number
represented by the difference between the number supplied and the number demanded at the minimum wage.

(b) The demand for labor is derived from the demand for the products labor produces. An increase in the demand
for goods produced in Bazra will increase the demand for labor in Bazra. Whether or not the increase in
labor demand leads to an equilibrium wage above the minimum wage, the number of workers employed
will increase.

Grading Rubric:

Question 3: 9 points (4 + 3 + 2)

(a) 4 points

1 point: correctly labeled graph without regard to the shift


1 point: shifting the labor demand curve to the left
1 point: (i) wage rate and the number employed would fall
1 point: (ii) number of unemployed workers is equal to zero

Copyright © 2002 by College Entrance Examination Board. All rights reserved.


Advanced Placement Program and AP are registered trademarks of the College Entrance Examination Board.

6
AP® MICROECONOMICS
2002 SCORING GUIDELINES (Form B)

Question 3 (cont’d.)

(b) 3 points

1 point: the wage rate will rise


1 point: the number or workers employed will fall
1 point: the number of unemployed workers will rise

(c) 2 points (1 for the direction and 1 for the explanation)

1 point: employment increases


1 point: the increased demand for goods causes the (derived) demand for labor to
increase, shifting the the demand curve to the right.

Copyright © 2002 by College Entrance Examination Board. All rights reserved.


Advanced Placement Program and AP are registered trademarks of the College Entrance Examination Board.

7
AP® MICROECONOMICS
2003 SCORING GUIDELINES

Question 1

Correct Answers:

Part a: The market graph should have a downward-sloping demand curve and an upward -
sloping supply curve with an equilibrium price and quantity clearly labeled. The firm graph
should have a perfectly elastic (or horizontal) demand curve at the equilibrium market price. The
firm’s profit-maximizing quantity is found at the intersection of this demand or marginal revenue
curve with the firm’s marginal cost curve.

Part b: The firm’s profits are represented by the rectangle that has a height (or vertical distance)
of (P-ATC) multiplied by the firm’s profit-maximizing output or q.

Part c: With profits being earned, new firms will enter the smoke alarm market. The market
supply will increase (shift out to the right) and the equilibrium price will fall and quantity will
increase. As the market price falls, the firm has a downward shift in its horizontal demand curve.
The process continues until price of output has fallen to the minimum of the average total cost of
the firm.

Part d: With a positive consumption externality in the market for smoke alarms, the demand
curve with marginal social benefits should lie above the demand curve with only marginal private
benefits. Thus, the socially optimal output level will exceed the output level produced by an
unregulated private market.

Part e: To increase the market output to the socially optimal output, the government could
subsidize the consumption or production of smoke alarms.

Copyright © 2003 by College Entrance Examination Board. All rights reserved.


Available at apcentral.collegeboard.com.

2
AP® MICROECONOMICS
2003 SCORING GUIDELINES

Question 1 (cont’d)

Grading Rubric:
Point allocations: (12 points: 4+1+4+2+1)
(a) 4 points:
1 point for the market graph (S, D) with a downward-sloping demand curve and an
upward sloping supply curve.
1 point for correctly labeling equilibrium P and Q for the market.
1 point for the firm graph (Horizontal D or P curve).
1 point for applying MR=MC to find equilibrium quantity.
• MR must be logically consistent with demand curve

(b) 1 point for showing the AREA of economic profit for the firm.
• Must use P, ATC, and q.
(c) 4 points: (2 +2)
(i) 2 points:
1 point for showing an increase in supply on market graph
(resulting from the entry of new firms).
1 point for showing both a lower P and higher Q due to an increase in
supply.
(ii) 2 points:
1 point for the downward shift in the firm’s demand curve (P or MR or D)
1 point for q (for firm) where P = min ATC for firm

Copyright © 2003 by College Entrance Examination Board. All rights reserved.


Available at apcentral.collegeboard.com.

3
AP® MICROECONOMICS
2003 SCORING GUIDELINES

Question 1 (cont’d)

_______________________________________________________
(d) 2 points:

1 point for showing that Qs > Qm.


1 point for having two marginal benefit curves: one with and without the
positive externality.

(e) 1 point for any of the following: Subsidize sellers or buyers, mandatory smoke alarm system,
or tax relief.

Copyright © 2003 by College Entrance Examination Board. All rights reserved.


Available at apcentral.collegeboard.com.

4
AP® MICROECONOMICS
2003 SCORING GUIDELINES

Question 2

Correct Answers:

Part a: For the monopolist, a correctly labeled graph should show a downward-sloping demand
curve with a marginal revenue curve that lies below the demand curve. The monopolist’s profit-
maximizing output is found at the intersection of marginal revenue and marginal cost. The price
is found on the demand curve, above the quantity produced. The firm’s profits are represented by
the rectangle that has a height (or vertical distance) of (P-ATC) multiplied by the profit-
maximizing output or Q.

Part b: Marginal revenue is less than price since to sell additional units of output, the monopolist
must lower price on all units of output sold.

Part c: Consumer surplus is the area bounded vertically by the difference between the demand
curve (willingness to pay) and the monopolist’s price over the number of units sold by the
monopolist. The deadweight loss from monopoly is the combination of consumer and producer
surplus that is lost when comparing the monopoly output to the output that would be produced
under competitive conditions (where P=MC).

Copyright © 2003 by College Entrance Examination Board. All rights reserved.


Available at apcentral.collegeboard.com.

5
AP® MICROECONOMICS
2003 SCORING GUIDELINES

Question 2 (cont’d)

Grading Rubric:
Point allocations: (7 points: 4 + 1 + 2)
(a) 4 points
1 point for correctly labeled graph with downward-sloping demand curve AND
marginal revenue curve below demand.
1 point for indicating Q at MR=MC.
1 point for finding the appropriate P on the demand curve directly above
MR=MC output.
1 point for area of profit (must use P, ATC, and Q).

(b) 1 point - the monopolist must lower its price on all units to sell addtional
quantities so MR<P.

(c) 2 points
1 point for indicating correct area for consumer surplus.
1 point for indicating correct area for deadweight loss.

Copyright © 2003 by College Entrance Examination Board. All rights reserved.


Available at apcentral.collegeboard.com.

6
AP® MICROECONOMICS
2003 SCORING GUIDELINES

Question 3

Correct Answers:

Part a: The marginal revenue product of labor is the change in total revenue associated with the
change in output following a unit change in the employment of labor.

MRP of labor = MR (or P of output) x MPP of labor.

Part b: The perfectly competitive labor market will have a downward-sloping labor demand
curve and an upward-sloping labor supply curve. There will be an equilibrium wage and quantity
of labor. The firm will be a wage taker and have a perfectly elastic labor supply at the market
wage rate. The firm’s labor demand curve is its marginal revenue product of labor curve. Thus,
the profit-maximizing firm will hire the amount of labor associated with the intersection of its
marginal revenue product of labor curve and its horizontal labor supply curve. The firm will pay
each unit of labor the market wage.

Part c: With an increase in labor productivity that affects only Company XYZ there will be no
perceptible change in labor market. The equilibrium wage stays the same. Company XYZ will
have an outward shift in its marginal revenue product of labor (or labor demand) curve, leading to
more employment at the unchanged market wage.

Copyright © 2003 by College Entrance Examination Board. All rights reserved.


Available at apcentral.collegeboard.com.

7
AP® MICROECONOMICS
2003 SCORING GUIDELINES

Question 3 (cont’d)

Grading Rubric:
Point allocations: (7 points: 1+4+2)
(a) 1 point - definition: the additional revenue from hiring an additional worker or
MRP=MP x MR or MRP=MP x P
(b) 4 points:

1 point for the correctly labeled labor market graph showing equilibrium
wage.
1 point for the firm graph indicating a downward-sloping demand (MRPL)
curve.
1 point for the horizontal labor supply curve for the firm.
1 point for showing quantity of labor for the firm at the intersection of labor
supply and labor demand.

Copyright © 2003 by College Entrance Examination Board. All rights reserved.


Available at apcentral.collegeboard.com.

8
AP® MICROECONOMICS
2003 SCORING GUIDELINES

Question 3 (cont’d)

(c) (2 points)
1 point - wage rate for the firm remains constant at the original wage.
1 point - the number of worker hired by XYZ increases because XYZ’s
MRP increases or labor demand curve shifts out.

Copyright © 2003 by College Entrance Examination Board. All rights reserved.


Available at apcentral.collegeboard.com.

9
AP® MICROECONOMICS
2003 SCORING GUIDELINES (Form B)

Question 1

Correct Answers:

(a) The diagram above illustrates the graphs for a monopoly and a competitive firm earning
short-run economic profits. The areas of economic profits are shaded with horizontal
lines.

(b) The competitive firm is a price taker with a horizontal demand curve determined by the
market equilibrium price. It can sell as many units as it wants at the market price, so it
takes that price as its marginal revenue for each good. Price and marginal revenue are
therefore the same. The monopoly faces a downward-sloping demand curve, meaning
that it must lower its price in order to sell more units. When it lowers its price to sell
another unit, its marginal revenue is the new price minus the loss it incurs by lowering its
price on units previously sold at a higher price. Thus, marginal revenue is below price for
a monopoly.

(c) Under perfect competition there are no barriers to entry, so in the long run new firms will
enter when economic profits are being made. The entry of new firms increases market
supply and lowers market price until the price falls to the level of minimum average total
cost and zero economic profits are being made. Barriers to entry allow the monopoly to
continue to earn economic profits even in the long run.

(d) The triangular shaded area in the monopoly graph represents the deadweight loss caused
by the monopoly firm. The deadweight loss represents the loss of consumer and producer
surplus caused by a suboptimal level of production at which marginal cost is below the
price level.

Copyright © 2003 by College Entrance Examination Board. All rights reserved.


Available at apcentral.collegeboard.com.

2
AP® MICROECONOMICS
2003 SCORING GUIDELINES (Form B)

Question 1 (cont’d)

Grading Rubric:

Question 1: 14 points

a) 8 points
Monopoly Perfectly Competitive Firm
1 - Graph with downward D and 1 - Graph with horizontal D
MR below D
1 - P from the demand curve 1 - P at intersection of D and MC/S
1- Q at MR=MC 1 - Q at MR=MC
1- Area of profit 1 - Area of profit

b) 2 points: (½ for each relationship and ½ for each explanation;)


½ point - P is the same as MR.
½ point - Because P is constant / firm is a price taker.
½ point - MR < P.
½ point - Because the firm must lower price (on all units) to sell more output.

c) 2 points: (one for competitive story and one for monopoly story)

1 point – With perfect competition, the entry of new firms increases supply and
decreases price to the level of minimum average total cost, resulting in zero
economic profits.
1point – For the monopoly, economic profits can continue in the long run due to
barriers to entry.
d) 2 points:
1 point - Correct area, including lost consumer surplus and lost producer surplus.
1 point - Lost consumer and producer surplus due to suboptimal quantity/production
where P>MC.

Copyright © 2003 by College Entrance Examination Board. All rights reserved.


Available at apcentral.collegeboard.com.

3
AP® MICROECONOMICS
2003 SCORING GUIDELINES (Form B)

Question 2

Correct Answers:

(a) Before trade, the relevant curves are the domestic supply curve and the demand curve.
The intersection of domestic supply and demand determines the equilibrium price and
quantity, P1 and Q1. Consumer surplus is area H, above the price line and below the
demand curve. Producer surplus is area I + L + T, above the domestic supply curve and
below the price line.

(b) With free trade and a world price of PW, domestic suppliers are unable to charge more
than PW because consumers can purchase all that they want at the world price. At PW,
domestic suppliers are willing to supply quantity Q1 and demanders will purchase
quantity Q5. The difference between these quantities, Q5 – Q1, will be imported by
Country X.

(c) At the post-tariff price PT, the domestic supply curve indicates that domestic suppliers
will produce the quantity Q2 rather than Q1, so the change is Q2 – Q1. The demand curve
indicates that domestic consumers will demand the quantity Q4 rather than Q5, so the
change is Q4 – Q5. Consumer surplus shrinks from (H+I+J+K+L+M+N+R+S) to
(H+I+J+K), a loss of (L+M+N+R+S). Producer surplus increases from T to T+L, a gain
of L.

Copyright © 2003 by College Entrance Examination Board. All rights reserved.


Available at apcentral.collegeboard.com.

4
AP® MICROECONOMICS
2003 SCORING GUIDELINES (Form B)

Question 2 (cont’d)

Grading Rubric:

Question 1: 8 points

a) 3 points:
1 point - for identifying Q3 and P1
1 point - for identifying H
1 point - for identifying I+L+T

b) 1 point - for identifying Q5-Q1

c) 4 points
1 for identifying Q2-Q1; Q2 is acceptable if part (b) is correct.
1 for identifying Q4-Q5; Q4 is acceptable if part (b) is correct.
1 for identifying the loss: L+M+N+R+S; (retain H+I+J+K not acceptable—must identify the
CHANGE).
1 for identifying the gain: L; (T + L not acceptable—must identify the CHANGE).

Copyright © 2003 by College Entrance Examination Board. All rights reserved.


Available at apcentral.collegeboard.com.

5
AP® MICROECONOMICS
2003 SCORING GUIDELINES (Form B)

Question 3

Correct Answers:

(a) A profit-maximizing firm will hire workers until the marginal revenue product equals the
marginal factor cost.

(b) The marginal revenue product is determined by multiplying the marginal product of
workers by the price of pencils ($2). Calculated in this way, the marginal revenue
product for the first six workers is $16, $14, $12, $10, $8, and $6. The marginal factor
cost for a firm like this one that can hire all the workers that it wants at a fixed wage is
equivalent to that wage, which in this case is $8. The profit maximizing condition that
marginal revenue product equals the wage rate is met when five workers are hired,
making five the profit-maximizing number of workers. Note that the 5th worker costs the
firm $8 and contributes $8 worth of output, so the firm is in fact indifferent between
hiring four or five workers.

(c) With a wage rate of $6, the firm would do well to hire 6 workers, because it is with 6
workers that the marginal revenue product equals the new wage. Since the 6th worker
contributes $6 worth of output and costs $6, it is also possible that the firm will decide to
stop hiring after the 5th worker.

(d) With a pencil price of $1, the marginal revenue product for the first six workers is 8, 7, 6,
5, 4, 3, 2, and 1. Marginal revenue product equals wage when three workers are hired,
meaning that profits will be maximized with two or three workers (the firm is indifferent
between hiring the third or not).

Copyright © 2003 by College Entrance Examination Board. All rights reserved.


Available at apcentral.collegeboard.com.

6
AP® MICROECONOMICS
2003 SCORING GUIDELINES (Form B)

Question 3 (cont’d)

Grading Rubric:

Question 3: 5 points

a) 1 point: Marginal revenue product (MRP) = Marginal factor cost (MFC)


Acceptable answers include: MRP = MFC, MRP =W, MPL x MR = W, additional
revenue from one more worker = wage, but not MR = W.

b) 2 points:
1 point - 4 or 5 workers
1 point - Hire until MRP = $8 = W

c) 1 point - 5 or 6 workers

d) 1 point: 2 or 3 workers

Copyright © 2003 by College Entrance Examination Board. All rights reserved.


Available at apcentral.collegeboard.com.

7
AP® MICROECONOMICS
2004 SCORING GUIDELINES

Question 1

Correct Answer:
(a) As shown on the graph, MSC>MPC indicates the existence of a negative externality.
(b) The socially optimal quantity is Q2 because MSB=MSC at this output level.
(c) The profit-maximizing quantity is Q1 because MR=MPC at this output level. To produce the socially
optimum quantity, the government should grant the monopolist a per-unit subsidy of $3.00.
(d) The equilibrium quantity for the perfectly competitive industry is Q3 because MPC =D(MSB) at this
output level. To produce the socially optimal quantity, the government should levy a per-unit tax of
$5.00 on firms in this industry.

Scoring Guidelines: 12 points (2+2+4+4)


(a) 2 points:
1 - Identifying negative externality
1 - (Marginal) social cost is higher than (marginal) private cost, (M)SC>(M)PC
**Contingent on negative externality as correct assertion

(b) 2 points:
1 - Identifying Q2
1 - Marginal social benefit equals marginal social cost (D=MSC or P=MSC)

(c) 4 points:
(i) 1 - Identifying Q1
1 - At Q1, marginal private cost equals marginal revenue (MPC=MR or MC=MR)
(ii) 1 - Subsidize the firm
(iii) 1 - $3 per unit
**Contingent on subsidy as correct assertion in (c)(ii)

(d) 4 points:
(i) 1 - Identifying Q3
1 - Marginal social benefit equals marginal private cost
D=MPC or MSB=MPC
(ii) 1 - Tax the industry
(iii) 1 - $5 per unit
** Contingent on tax as correct assertion in (d)(ii)

Copyright © 2004 by College Entrance Examination Board. All rights reserved.


Visit apcentral.collegeboard.com (for AP professionals) and www.collegeboard.com/apstudents (for AP students and parents).

2
AP® MICROECONOMICS
2004 SCORING GUIDELINES

Question 2

Correct Answer:
(a) As shown on the graph, P2 and Q2 were the price and quantity of oil before trade in the US market.
(b) The amount of oil imported into the US market after trade would be equal to Q3-Q1. US production
drops to Q1 but quantity demanded rises to Q3.
(c) The triangle P2KG represents consumer surplus before trade, while triangle PWKH represents consumer
surplus after trade.
(d) The triangle P1P2G represents producer surplus before trade, while triangle P1PWJ represents producer
surplus after trade.
(e) The triangle JGH shows the net gain in total surplus from trade.

Scoring Guidelines: 8 points (2+1+2+2+1)


(a) 2 points: (Pre-trade)
1 - Identifying P2
1 - Identifying Q2

(b) 1 point: (Imports) Identifying (Q3 - Q1) or (H-J)

(c) 2 points: (Consumer surpluses)


1 - Identifying P2KG (before)
1 - Identifying PWKH (after)

(d) 2 points: (Producer surpluses)


1 - Identifying P1P2G (before)
1 - Identifying P1PWJ (after)

(e) 1 point: (Net gain in surplus) Identifying JGH

Copyright © 2004 by College Entrance Examination Board. All rights reserved.


Visit apcentral.collegeboard.com (for AP professionals) and www.collegeboard.com/apstudents (for AP students and parents).

3
AP® MICROECONOMICS
2004 SCORING GUIDELINES

Question 3

Correct Answer:
(a) The correct graph for a monopolistically competitive firm will show a downward-sloping D curve
with a downward-sloping MR curve below it. The firm’s price and output would be found at the
equality of MR and MC. In the long run, the ATC curve is tangent to the demand curve and equal to
price directly above the output level at which MR=MC.
(b) When the fixed cost decreases, MC is not affected so that the output and price remain constant.
Economic profit increases since the ATC falls.

Scoring Guidelines: 8 points (4+4)

(a) 4 points:
1 - Graph with a downward-sloping demand curve with correctly labeled axes
1 - Downward-sloping marginal revenue curve below the demand curve
1 - Q from MR = MC and P from Demand directly above Q
1 - Long-run equilibrium, AC (or ATC) tangent to Demand at Q
(b) 4 points:
1 - Individual firm’s output level does not change
1 – License fee is a fixed cost, thus it does not affect the firm’s marginal cost
1 - Economic profits increase
1 - Explanation

**Contingent on b(i)
If Q does not change in b(i), TC or ATC decrease is sufficient.
If Q changes in b(i), then “correct” explanation of how TR/TC or AR/ATC is affected
is necessary.

Copyright © 2004 by College Entrance Examination Board. All rights reserved.


Visit apcentral.collegeboard.com (for AP professionals) and www.collegeboard.com/apstudents (for AP students and parents).

4
AP® MICROECONOMICS
2004 SCORING GUIDELINES (Form B)

Question 1

9 points (1+4+2+1+1)
(a) 1 point: Brunelle must lower its price on all units to sell additional units. Thus, the additional revenue
from the last unit sold is the price minus the loss on units that would otherwise sell at a higher
price.

(b) 4 points:
1 – Correctly labeled graph with downward-sloping demand and marginal revenue below demand
1 - For Q* at MC=MR
1 - For P* at the height of the demand curve above MC=MR
1 - For shading the correct area of profit (P*-ATC)Q*

(c) 2 points:
1 - For Identifying Qr at MR =0
1 - For identifying Pr at the height of the demand curve above Qr

(d) 1 point: Brunelle is not producing the allocatively efficient level of output because P>MC (MSB > MSC)

(e) 1 point: Brunell’s demand curve will shift inward to the left

Copyright © 2004 by College Entrance Examination Board. All rights reserved.


Visit apcentral.collegeboard.com (for AP professionals) and www.collegeboard.com/apstudents (for AP students and parents).

2
AP® MICROECONOMICS
2004 SCORING GUIDELINES (Form B)

Question 2

6 points (2+2+2)
(a) 2 points:

1 - Perfectly inelastic demand curve showing that Q does not change


1 – Since producers can raise the price by the full amount of the tax, the tax falls entirely on buyers
(b) 2 points:

1 – Horizontal demand curve showing that price does not change


1 – The tax falls entirely on sellers, since they can’t charge more and thus must absorb the entire
amount of the tax
(c) 2 points:

1 - For shifting either the supply curve or the demand curve inward to the left
1 - For shading the correct profits area
Copyright © 2004 by College Entrance Examination Board. All rights reserved.
Visit apcentral.collegeboard.com (for AP professionals) and www.collegeboard.com/apstudents (for AP students and parents).

3
AP® MICROECONOMICS
2004 SCORING GUIDELINES (Form B)

Question 3

6 points (2+1+2+1)

(a) 2 points:
1 – Placonia imports the good
1 – The domestic opportunity cost of producing good X is higher than the world price (PW) for unit
JN. Or, they can get it cheaper at the world price.

(b) 1 point: They import 300 (=350 - 50) units or JN units

(c) 2 points:
1 – Consumer surplus decreases from PwNH to PtMH, or a decrease of MNPWPt
1 – Producer surplus increases by JKPtPW

(d) 1 point: Employment would increase because domestic production of good X increases in Placonia

Copyright © 2004 by College Entrance Examination Board. All rights reserved.


Visit apcentral.collegeboard.com (for AP professionals) and www.collegeboard.com/apstudents (for AP students and parents).

4
AP® MICROECONOMICS
2005 SCORING GUIDELINES
Question 1

12 points (4 + 4 + 1 + 3)

(a) 4 points:
• One point is earned for a correctly labeled market graph of the dairy industry.
• One point is earned for correctly labeled equilibrium industry price and quantity.
• One point is earned for drawing a horizontal demand curve for Bestmilk at the market price.
• One point is earned for showing the equilibrium quantity for Bestmilk where price equals marginal
cost and the minimum of average total cost (P=MC=ATC).

(b) 4 points:
• One point is earned for showing a decrease in market demand.
• One point is earned for showing a decrease in equilibrium market price and quantity.
• One point is earned for showing the change to a new lower profit-maximizing price and quantity
for Bestmilk.
• One point is earned for shading the area of loss for Bestmilk.

(c) 1 point:
• One point is earned for stating that P ≥ AVC, or TR ≥ TVC, or P>AVC, or TR>TVC, or losses are less
than total fixed cost.

(d) 3 points:
• One point is earned for stating that industry price returns to the original long-run equilibrium price.
• One point is earned for stating that the output of a typical firm returns to the original profit-
maximizing quantity.
• One point is earned for stating that there is a decrease in the number of firms.

Copyright © 2005 by College Board. All rights reserved.


Visit apcentral.collegeboard.com (for AP professionals) and www.collegeboard.com/apstudents (for AP students and parents).

2
AP® MICROECONOMICS
2005 SCORING GUIDELINES
Question 2

9 points (3 + 2 + 4)

(a) 3 points:
• One point is earned for identifying P=$12 and Q=100 units.
• One point is earned for identifying the consumer surplus before the tax, A+B+C+F.
• One point is earned for identifying the producer surplus before the tax, D+E+G or $100.

(b) 2 points:
• One point is earned for stating that the price paid by the buyers does not rise by the full amount of
the tax.
• One point is earned for a correct explanation: P increases by $1 and the tax is $2 per unit; or a
correct elasticity explanation, such as S is not perfectly elastic, or D is not perfectly inelastic, or D
and S have the same elasticities.

(c) 4 points:
• One point is earned for identifying the net price received by the sellers, $11.
• One point is earned for identifying the tax revenue, B+C+D or $160.
• One point is earned for identifying consumer surplus, A.
• One point is earned for identifying the deadweight loss, F+G or $20.

Copyright © 2005 by College Board. All rights reserved.


Visit apcentral.collegeboard.com (for AP professionals) and www.collegeboard.com/apstudents (for AP students and parents).

3
AP® MICROECONOMICS
2005 SCORING GUIDELINES
Question 3

8 points (2 + 2 + 2 + 2)

(a) 2 points:
• One point is earned for indicating perfect competition.
• One point is earned for a correct explanation: The price is constant, or the firm is a price taker in
the product market.

(b) 2 points:
• One point is earned for indicating perfect competition.
• One point is earned for a correct explanation: The wage rate is constant, or the firm is a price taker
in the labor market.

(c) 2 points:
• One point is earned for stating that MRP is $400.
• One point is earned for showing the calculation: MRP = P x MP, so MRP = 20 x $20 = $400

(d) 2 points:
• One point is earned for indicating six workers, or in between six and seven workers.
• One point is earned for a correct explanation: MRP>W for the sixth worker, but MRP<W for the
seventh worker.

Copyright © 2005 by College Board. All rights reserved.


Visit apcentral.collegeboard.com (for AP professionals) and www.collegeboard.com/apstudents (for AP students and parents).

4
AP® MICROECONOMICS
2005 SCORING GUIDELINES (Form B)
Question 1

13 points (4 + 4 + 2 + 3)

(a) 4 points:
• One point is earned for a graph with downward sloping demand curve with MR below.
• One point is earned for showing output at MR=MC.
• One point is earned for showing price on the demand curve above MR=MC.
• One point is earned for showing the area of profit, shaded or labeled.

(b) 4 points:
• One point is earned for stating that MRP=MFC or MRP=Wage or MRP=MRC.
• One point is earned for drawing a correct graph with downward sloping labor-demand curve.
• One point is earned for drawing a horizontal labor-supply curve at the wage rate.
• One point is earned for indicating the profit-maximizing quantity of labor.

(c) 2 points:
• One point is earned for showing that labor-supply curve shifts down.
• One point is earned for showing that the amount of labor hired increases.

Copyright © 2005 by College Board. All rights reserved.


Visit apcentral.collegeboard.com (for AP professionals) and www.collegeboard.com/apstudents (for AP students and parents).

2
AP® MICROECONOMICS
2005 SCORING GUIDELINES (Form B)
Question 1 (continued)

(d) 3 points:
• One point is earned for indicating that the total fixed cost would not change.
• One point is earned for indicating that the marginal cost would fall.
• One point is earned for indicating that the product price would fall.

Copyright © 2005 by College Board. All rights reserved.


Visit apcentral.collegeboard.com (for AP professionals) and www.collegeboard.com/apstudents (for AP students and parents).

3
AP® MICROECONOMICS
2005 SCORING GUIDELINES (Form B)
Question 2

8 points (3 + 2 + 1 + 1 + 1)

(a) 3 points:
• One point is earned for identifying curve I as marginal cost (MC).
• One point is earned for identifying curve II as average total cost (ATC).
• One point is earned for identifying curve III as average variable cost (AVC).

(b) 2 points:
• One point is earned for a correct explanation for (i): increasing marginal returns or increasing
marginal product (productivity) or increasing returns or specialization.
• One point is earned for a correct explanation for (ii): decreasing marginal returns (productivity) or
(the law of) diminishing marginal returns or sharing of fixed inputs.

(c) 1 point:
• One point is earned for identifying the average fixed cost.

(d) 1 point:
• One point is earned for stating that the fixed cost is spread over larger quantity, so AFC decreases.

(e) 1 point:
• One point is earned for identifying K and L.

Copyright © 2005 by College Board. All rights reserved.


Visit apcentral.collegeboard.com (for AP professionals) and www.collegeboard.com/apstudents (for AP students and parents).

4
AP® MICROECONOMICS
2005 SCORING GUIDELINES (Form B)
Question 3

5 points (4 + 1)

(a) 4 points:
• One point is earned for correctly labeled supply and demand graphs for all three markets.
• One point is earned for showing a rightward shift of the supply curve in the wheat market with
price decreasing and quantity increasing.
• One point is earned for showing a rightward shift of the supply curve in the bread market with
price decreasing and quantity increasing.
• One point is earned for showing a rightward shift of the demand curve in the butter market with
price increasing and quantity increasing.

(b) 1 point:
• One point is earned for concluding that the total revenue will increase.

Copyright © 2005 by College Board. All rights reserved.


Visit apcentral.collegeboard.com (for AP professionals) and www.collegeboard.com/apstudents (for AP students and parents).

5
AP® MICROECONOMICS
2006 SCORING GUIDELINES

Question 1

11 points (4 + 2 + 2 + 3)

(a) 4 points:
• One point is earned for indicating P5, Q2.
• One point is earned for indicating P3, Q4.
• One point is earned for indicating P4, Q3.
• One point is earned for indicating P2, Q5.

(b) 2 points:
• One point is earned for stating that demand is elastic at Q1.
• One point is earned for explaining that MR is greater than zero, OR Q1 is to the left of
the midpoint, OR Q1 is in the upper half of the demand curve.

(c) 2 points:
• One point is earned for stating that accounting profits are positive.
• One point is earned for explaining that economic profits are zero, opportunity costs
exist, and economic profits=accounting profits-opportunity cost.

(d) 3 points:
• One point is earned for indicating Q7.
• One point is earned for concluding that the outcome is not allocatively efficient.
• One point is earned for explaining that MC>P or MSC>MSB.

© 2006 The College Board. All rights reserved.


Visit apcentral.collegeboard.com (for AP professionals) and www.collegeboard.com/apstudents (for students and parents).

2
AP® MICROECONOMICS
2006 SCORING GUIDELINES

Question 2

7 points (1 + 1 + 2 + 2 + 1)

(a) 1 point:
• One point is earned for indicating that TFC is $20.

(b) 1 point:
• One point is earned for indicating that MC of the first unit is $7.

(c) 2 points:
• One point is earned for indicating that the profit-maximizing output=4 units (or
between 4 and 5 units).
• One point is earned for explaining that MR>MC for all units until Q=5 (or direct
calculation of TR-TC).

(d) 2 points:
• One point is earned for concluding that the number of firms will increase.
• One point is earned for explaining that profits will attract new firms to enter.

(e) 1 point:
• One point is earned for stating that there is no change in the profit-maximizing output.

© 2006 The College Board. All rights reserved.


Visit apcentral.collegeboard.com (for AP professionals) and www.collegeboard.com/apstudents (for students and parents).

3
AP® MICROECONOMICS
2006 SCORING GUIDELINES

Question 3

6 points (2 + 2 + 2)

(a) 2 points:
• One point is earned for showing a rightward shift of the demand curve.
• One point is earned for showing that equilibrium price increases and quantity
increases.

(b) 2 points:
• One point is earned for showing a rightward shift of the supply curve.
• One point is earned for showing that equilibrium price decreases and quantity
increases.

(c) 2 points:
• One point is earned for indicating that MSC is greater than MPC.
• One point is earned for stating that the conversion of land to residential development is
not socially optimum because MSC>MSB (P).

© 2006 The College Board. All rights reserved.


Visit apcentral.collegeboard.com (for AP professionals) and www.collegeboard.com/apstudents (for students and parents).

4
AP® MICROECONOMICS
2006 SCORING GUIDELINES (Form B)

Question 1

11 points (1 + 4 + 2 + 4)

(a) 1 point:
• One point is earned for stating that the firm must be covering its AVC (or TVC), or P>AVC.

(b) 4 points:
• One point is earned for a correctly labeled graph with MR below the demand curve.
• One point is earned for identifying profit-maximizing quantity at MR=MC.
• One point is earned for identifying price on the demand curve above equilibrium quantity and
below ATC.
• One point is earned for showing the correct loss area.

(c) 2 points:
• One point is earned for indicating that total revenue will fall.
• One point is earned for explaining that demand is elastic or MR is positive.

(d) 4 points:
• One point is earned for indicating that the profit-maximizing output will increase.
• One point is earned for explaining that the marginal revenue curve will shift to the right.
• One point is earned for concluding that total cost will increase.
• One point is earned for explaining that output increases.

© 2006 The College Board. All rights reserved.


Visit apcentral.collegeboard.com (for AP professionals) and www.collegeboard.com/apstudents (for students and parents).

2
AP® MICROECONOMICS
2006 SCORING GUIDELINES (Form B)

Question 2

7 points (3 + 2 + 2)

(a) 3 points:
• One point is earned for a correctly labeled graph with equilibrium price and quantity.
• One point is earned for showing the correct area of consumer surplus.
• One point is earned for showing the correct area of producer surplus.

(b) 2 points:
• One point is earned for showing the price ceiling below the equilibrium.
• One point is earned for shading the correct area of consumer surplus and producer surplus.

(c) 2 points:
• One point is earned for indicating that consumer surplus will decrease.
• One point is earned for indicating that producer surplus will not change.

© 2006 The College Board. All rights reserved.


Visit apcentral.collegeboard.com (for AP professionals) and www.collegeboard.com/apstudents (for students and parents).

3
AP® MICROECONOMICS
2006 SCORING GUIDELINES (Form B)

Question 3

7 points (1 + 1 + 3 + 2)

(a) 1 point:
• One point is earned for stating that MRP=MFC.

(b) 1 point:
• One point is earned for calculating the price: $80/20= $4

(c) 3 points:
• One point is earned for a correctly labeled graph with downward-sloping demand curve.
• One point is earned for drawing a horizontal supply curve.
• One point is earned for showing equilibrium amount of labor.

(d) 2 points:
• One point is earned for stating that the amount of labor will increase.
• One point is earned for explaining that MRP>W.

© 2006 The College Board. All rights reserved.


Visit apcentral.collegeboard.com (for AP professionals) and www.collegeboard.com/apstudents (for students and parents).

4
AP® MICROECONOMICS
2007 SCORING GUIDELINES

Question 1

12 points (4 + 3 + 3 + 2)

(a) 4 points:
• One point is earned for correctly labeled axes and an MR curve below a downward-
sloping demand curve.
• One point is earned for showing profit-maximizing Q at MC=MR.
• One point is earned for identifying P on the demand curve above Q.
• One point is earned for showing that P>ATC at Q.

(b) 3 points:
• One point is earned for concluding that profit-maximizing Q and P will not change.
• One point is earned for correctly explaining that the lump-sum tax will not affect
MC.
• One point is earned for concluding that profits will decrease.

(c) 3 points:
• One point is earned for concluding that profit-maximizing Q will increase and P will
decrease.
• One point is earned for explaining that the MC curve shifts down.
• One point is earned for concluding that profits will increase.

(d) 2 points:
• One point is earned for concluding that GCR’s profits will fall in the long run.
• One point is earned for stating that new firms will enter the market.

© 2007 The College Board. All rights reserved.


Visit apcentral.collegeboard.com (for AP professionals) and www.collegeboard.com/apstudents (for students and parents).
AP® MICROECONOMICS
2007 SCORING GUIDELINES

Question 2

6 points (1 + 2 + 3)

(a) 1 point:
• One point is earned for correctly labeled axes with a horizontal labor supply curve at
$90.

(b) 2 points:
• One point is earned for identifying the profit-maximizing output as 75, or between 75
and 79.
• One point is earned for stating that MRP is greater than $90 for the fifth worker, but less
than $90 for the sixth worker.

(c) 3 points:
• One point is earned for stating that the quantity of labor HZRad hires increases.
• One point is earned for the explanation that the marginal product of labor increases at
each input level, or the marginal product of labor curve shifts to the right, or the
demand curve for labor shifts to the right.
• One point is earned for stating that the wage rate will remain constant.

© 2007 The College Board. All rights reserved.


Visit apcentral.collegeboard.com (for AP professionals) and www.collegeboard.com/apstudents (for students and parents).
AP® MICROECONOMICS
2007 SCORING GUIDELINES

Question 3

6 points (1 + 1 + 1 + 2 + 1)

(a) 1 point:
• One point is earned for identifying the market as an oligopoly, since there are only two
firms and their actions are mutually interdependent.

(b) 1 point:
• One point is earned for stating that Rankin Wheels will choose early departure.

(c) 1 point:
• One point is earned for stating that Roadway’s dominant strategy is early departure.

(d) 2 points:
• One point is earned for stating that early departure is not a dominant strategy for
Rankin Wheels.
• One point is earned for reasoning that if Roadway chooses a late departure, Rankin
Wheels is better off choosing a late departure.

(e) 1 point:
• One point is earned for identifying $900 as Rankin Wheels’ daily profit.

© 2007 The College Board. All rights reserved.


Visit apcentral.collegeboard.com (for AP professionals) and www.collegeboard.com/apstudents (for students and parents).
AP® MICROECONOMICS
2007 SCORING GUIDELINES (Form B)

Question 1

12 points (4 + 2 + 2 + 1 + 1 + 2)

(a) 4 points:
One point is earned for a correctly labeled graph with a downward-sloping demand
curve and a marginal revenue curve below the demand curve.
One point is earned for showing the profit-maximizing Q* at MC = MR.
One point is earned for P* on the demand curve above MC = MR.
One point is earned for showing the correct area of profit, (P* - ATC)Q*.

(b) 2 points:
One point is earned for stating that it is price elastic (or “No”).
One point is earned for the explanation that MR is positive so that TR rises if P is
decreased.

(c) 2 points:
One point is earned for stating that the demand curve for the typical firm would shift to
the left.
One point is earned for the explanation that the entry of new firms reduces the market
share of existing firms.

© 2007 The College Board. All rights reserved.


Visit apcentral.collegeboard.com (for AP professionals) and www.collegeboard.com/apstudents (for students and parents).
AP® MICROECONOMICS
2007 SCORING GUIDELINES (Form B)

Question 1 (continued)

(d) 1 point:
One point is earned for showing that long-run equilibrium occurs at the tangency of
ATC and the demand curve at the profit-maximizing quantity.

(e) 1 point:
One point is earned for stating “No.”

(f) 2 points:
One point is earned for stating “No.”
One point is earned for the explanation that at the long-run equilibrium, P > MC.

© 2007 The College Board. All rights reserved.


Visit apcentral.collegeboard.com (for AP professionals) and www.collegeboard.com/apstudents (for students and parents).
AP® MICROECONOMICS
2007 SCORING GUIDELINES (Form B)

Question 2

6 points (1 + 1 + 1 + 2 + 1)

(a) 1 point:
One point is earned for identifying the market as an oligopoly because there is mutual
interdependence—the behavior of each firm affects the other.

(b) 1 point:
One point is earned for stating that an evening departure will be best for Airtouch.

(c) 1 point:
One point is earned for stating that Winward’s dominant strategy is a morning
departure.

(d) 2 points:
One point is earned for stating that choosing an evening departure is not a dominant
strategy for Airtouch.
One point is earned for correctly reasoning that Airtouch does not have a dominant
strategy because its best payoff depends on Winward’s choice (OR, more specifically, if
Windward chooses a morning departure, Airtouch is best off choosing a morning
departure).

(e) 1 point:
One point is earned for identifying $700.

© 2007 The College Board. All rights reserved.


Visit apcentral.collegeboard.com (for AP professionals) and www.collegeboard.com/apstudents (for students and parents).
AP® MICROECONOMICS
2007 SCORING GUIDELINES (Form B)

Question 3

6 points (2 + 2 + 2)

(a) 2 points:
One point is earned for stating “false.”
One point is earned for explaining that the difference between ATC and AVC is AFC,
which decreases as a firm’s output increases.

(b) 2 points:
One point is earned for stating “false.”
One point is earned for the explanation that the profit-maximizing output occurs where
P = MC, which might be to the right or to the left of the lowest point on the ATC curve.

(c) 2 points:
One point is earned for stating “false.”
One point is earned for the explanation that if the firm shuts down in the short run, its
losses from operating must be larger than (or equal to) its total fixed costs OR price
must be less than AVC, so it is incurring a loss.

© 2007 The College Board. All rights reserved.


Visit apcentral.collegeboard.com (for AP professionals) and www.collegeboard.com/apstudents (for students and parents).
AP® MICROECONOMICS
2008 SCORING GUIDELINES

Question 1

12 points (4 + 4 + 4)

(a) 4 points:
• One point is earned for a correctly labeled graph of the apple market, with PM
and QM properly indicated.
• One point is earned for showing that the firm’s price equals the market price.
• One point is earned for the tangency of flat firm demand (PF) and ATC.
• One point is earned for QF where MR (P) = MC.

(b) 4 points:
• One point is earned for concluding that the lump-sum subsidy will have no impact on Callahan’s
output.
• One point is earned for explaining that the lump-sum subsidy will not affect MC (or MR).
• One point is earned for concluding that Callahan’s profit will increase.
• One point is earned for concluding that the number of firms in the industry will not change.

(c) 4 points:
• One point is earned for concluding that the number of firms in the industry will increase.
• One point is earned for explaining that the existence of profits attracts new firms.
• One point is earned for concluding that the price will fall.
• One point is earned for concluding that industry output will increase.

© 2008 The College Board. All rights reserved.


Visit the College Board on the Web: www.collegeboard.com.
AP® MICROECONOMICS
2008 SCORING GUIDELINES

Question 2

5 points (1 + 2 + 2)

(a) 1 point:
• One point is earned for defining marginal utility as the extra satisfaction received from
consuming an additional unit of a good or service.

(b) 2 points:
• One point is earned for concluding that Mandy should purchase more fudge and less coffee.
• One point is earned for explaining that the per dollar MU for fudge is greater than the per dollar
MU for coffee.

(c) 2 points:
• One point is earned for stating that the price elasticity of demand for good R is zero.
• One point is earned for stating that none of the tax will be paid by the seller of good R, or that
buyers pay all of the tax.

© 2008 The College Board. All rights reserved.


Visit the College Board on the Web: www.collegeboard.com.
AP® MICROECONOMICS
2008 SCORING GUIDELINES

Question 3

7 points (3 + 4)

(a) 3 points:
• One point is earned for a correctly labeled supply and demand graph with a price ceiling
indicated below the market equilibrium.
• One point is earned for correctly labeling QA.
• One point is earned for correctly labeling QB.

(b) 4 points:

(i) 2 points:
• One point is earned for identifying Q1 as the profit-maximizing output.
• One point is earned for identifying Q3 as the socially efficient output.

(ii) 2 points:
• One point is earned for recognizing that at Q3, the monopolist incurs a loss.
• One point is earned for identifying the area of loss, P1P3DF.

© 2008 The College Board. All rights reserved.


Visit the College Board on the Web: www.collegeboard.com.
AP® MICROECONOMICS
2008 SCORING GUIDELINES (Form B)

Question 1

10 points (2 + 2 + 1 + 3 + 1 + 1)

(a) 2 points:
• One point is earned for identifying the profit-maximizing output, Q2, and explaining that MC=MR
at Q2.
• One point is earned for identifying P5 on the demand curve above MC = MR.

(b) 2 points:
• One point is earned for identifying the revenue-maximizing output, Q3, and explaining that since
MR is zero at Q3, TR is at the maximum.
• One point is earned for identifying P3 as the price corresponding to Q3.

(c) 1 point:
• One point is earned for identifying the price the government would require, P1, and the allocatively
efficient output, Q4.

(d) 3 points:
• One point is earned for stating that the firm would incur a loss.
• One point is earned for the explanation that ATC is greater than the price, P1.
• One point is earned for identifying the area of loss, P1P2eg.

(e) 1 point:
• One point is earned for identifying the consumer surplus, P1P7g.

(f) 1 point:
• One point is earned for identifying the price, P3.

© 2008 The College Board. All rights reserved.


Visit the College Board on the Web: www.collegeboard.com.
AP® MICROECONOMICS
2008 SCORING GUIDELINES (Form B)

Question 2

6 points (3 + 1 + 2)

(a) 3 points:
• One point is earned for the correctly labeled graph with Qm and Pm correctly identified.
• One point is earned for correctly identifying QS.
• One point is earned for shading the area of the deadweight loss.

(b) 1 point:
• One point is earned for stating that the MSC is less than the marginal social benefit.

(c) 2 points:
• One point is earned for stating that the tax will increase the deadweight loss.
• One point is earned for the explanation that the tax raises the cost and causes output of vaccine to
fall.

© 2008 The College Board. All rights reserved.


Visit the College Board on the Web: www.collegeboard.com.
AP® MICROECONOMICS
2008 SCORING GUIDELINES (Form B)

Question 3

7 points (1 + 1 + 1 + 1 + 1 + 2)

(a) 1 point:
• One point is earned for stating the second worker.

(b) 1 point:
• One point is earned for stating the MP of the fifth worker is five units.

(c) 1 point:
• One point is earned for calculating the MRP of the third worker: $20.

(d) 1 point:
• One point is earned for stating the GW will hire four workers.

(e) 1 point:
• One point is earned for calculating the profit: $2.

(f) 2 points:
• One point is earned for stating that more workers will be hired.
• One point is earned for the explanation that the increase in the price of hats raises the marginal
revenue product, hence the demand for labor.

© 2008 The College Board. All rights reserved.


Visit the College Board on the Web: www.collegeboard.com.
AP® MICROECONOMICS
2009 SCORING GUIDELINES

Question 1

11 points (5 + 2 + 1 + 2 + 1)

(a) 5 points:
• One point is earned for a correctly labeled graph for CableNow, with a downward-sloping
demand curve and with the marginal revenue curve below the demand curve.
• One point is earned for identifying the profit-maximizing quantity of cable services, Q*, at
MC = MR.
• One point is earned for identifying the profit-maximizing price of cable services, P*, on the
demand curve above Q*.
• One point is earned for showing the area of economic profit, completely shaded.
• One point is earned for identifying the socially optimal level of cable services, QS, where the
MC curve intersects the demand curve.

(b) 2 points:
• One point is earned for stating that the lump-sum subsidy will have no impact on the quantity
of services CableNow produces.
• One point is earned for explaining that the lump-sum subsidy will not affect MC.

(c) 1 point:
• One point is earned for identifying the quantity of cable services, QR, where the ATC curve
intersects the demand curve.

(d) 2 points:
• One point is earned for stating that accounting profit is positive.
• One point is earned for explaining that accounting profit excludes implicit costs.

(e) 1 point:
• One point is earned for stating that the socially optimal quantity will be larger than QS.

© 2009 The College Board. All rights reserved.


Visit the College Board on the Web: www.collegeboard.com.
AP® MICROECONOMICS
2009 SCORING GUIDELINES

Question 2

6 points (1 + 3 + 1 + 1)

(a) 1 point:
• One point is earned for the correct calculation of the producer surplus: (1/2) × $3 × 90 = $135.

(b) 3 points:
• One point is earned for the correct calculation of the amount of tax revenue: $2 × 60 = $120.
• One point is earned for the correct calculation of the after-tax price received by sellers: $4.
• One point is earned for the correct calculation of the producer surplus: (1/2) × $2 × 60 = $60.

(c) 1 point:
• One point is earned for concluding that the demand price is elastic AND showing the correct
calculation of the elasticity coefficient using endpoint or midpoint method, or the correct
calculation using the total revenue formula.

(d) 1 point:
• One point is earned for concluding that, owing to the tax, the market is no longer allocatively
efficient AND that total surplus decreases or the tax creates a dead-weight loss.

© 2009 The College Board. All rights reserved.


Visit the College Board on the Web: www.collegeboard.com.
AP® MICROECONOMICS
2009 SCORING GUIDELINES

Question 3

6 points (2 + 2 + 1 + 1)

(a) 2 points:
• One point is earned for stating that north will be better for Blue Mart.
• One point is earned for explaining that Blue Mart earns a higher profit by locating north than it
does by locating south ($4,000 versus $1,000).

(b) 2 points:
• One point is earned for stating that choosing south is not a dominant strategy for Red Shop.
• One point is earned for explaining that if Blue Mart chooses south, Red Shop is better off
choosing north. (Red Shop’s best strategy depends on Blue Mart’s move.)

(c) 1 point:
• One point is earned for stating that Red Shop chooses south and Blue Mart chooses north.

(d) 1 point:
• One point is earned for redrawing the table with the correct entries:

Blue Mart
North South

$900, $1,800 $3,000, $5,500


North

Red Shop
$7,000, $4,000 $3,500, $3,000
South

© 2009 The College Board. All rights reserved.


Visit the College Board on the Web: www.collegeboard.com.
AP® MICROECONOMICS
2009 SCORING GUIDELINES (Form B)

Question 1

11 points (4 + 1 + 2 + 2 + 2)

(a) 4 points:
• One point is earned for a correctly labeled graph with a downward-sloping demand curve, with
MR below demand.
• One point is earned for identifying the profit-maximizing output, QM, at MC = MR.
• One point is earned for identifying PM on the demand curve above QM.
• One point is earned for showing the area of loss shaded completely.

(b) 1 point:
• One point is earned for stating that price is greater than average variable cost, or total revenue is
greater than total variable cost.

(c) 2 points:
• One point is earned for stating that the number of firms will increase.
• One point is earned for stating that economic profits will fall to zero or fall to a normal profit.

(d) 2 points:
• One point is earned for stating no.
• One point is earned for explaining that the firm’s price is greater than marginal cost.

(e) 2 points:
• One point is earned for stating yes.
• One point is earned for explaining that the firm produces a quantity of output in the declining
portion of its long-run ATC.

© 2009 The College Board. All rights reserved.


Visit the College Board on the Web: www.collegeboard.com.
AP® MICROECONOMICS
2009 SCORING GUIDELINES (Form B)

Question 2

6 points (2 + 2 + 2)

(a) 2 points:
• One point is earned for calculating the price of a unit of bananas, $4/8 = $0.50.
• One point is earned for stating that Sasha will purchase fewer peanuts.

(b) 2 points:
• One point is earned for stating that the equilibrium price and quantity of peanuts will both
increase.
• One point is earned for explaining that peanuts and bananas are substitutes, and since the price of
bananas increased, it would cause the demand for peanuts to increase.

(c) 2 points:
• One point is earned for stating that the substitution effect causes the quantity of bananas
demanded to decrease.
• One point is earned for stating that Sasha’s real income will decrease.

© 2009 The College Board. All rights reserved.


Visit the College Board on the Web: www.collegeboard.com.
AP® MICROECONOMICS
2009 SCORING GUIDELINES (Form B)

Question 3

6 points (1 + 2 + 1 + 1 + 1)

(a) 1 point:
• One point is earned for concluding that City Wheels maintains its current fare, since $180 > $120.

(b) 2 points:
• One point is earned for stating that Easy Ride does NOT have a dominant strategy.
• One point is earned for explaining that Easy Ride’s best move depends on City Wheels’ move.

(c) 1 point:
• One point is earned for stating that the profit to Easy Ride is $150 and the profit to City Wheels is
$180.

(d) 1 point:
• One point is earned for stating that the cooperative solution is for both to maintain their current
fares.

(e) 1 point:
• One point is earned for showing the correct entries in the new payoff matrix as follows:

City Wheels
Maintain Fare Lower Fare

$150, $180 $130, $160


Maintain Fare

Easy Ride
$160, $130 $180, $150
Lower Fare

© 2009 The College Board. All rights reserved.


Visit the College Board on the Web: www.collegeboard.com.
AP® MICROECONOMICS
2010 SCORING GUIDELINES

Question 1

10 points (4+1+4+1)

(a) 4 points:
• One point is earned for a correctly labeled graph of the corn market (S, D, PM1, QM1).
• One point is earned for the graph of the firm with a horizontal demand curve at PM1.
• One point is earned for showing the profit-maximizing quantity, QF1, at MC = MR.
• One point is earned for showing minimum ATC on the horizontal demand curve at QF1.

(b) 1 point:
• One point is earned for stating that the demand curve for Farmer Roy’s corn is perfectly elastic
because Farmer Roy is a price taker or because he can sell all that he wants at the market price.

(c) 4 points:
• One point is earned for shifting the market demand curve to the right and showing PM2 and QM2.
• One point is earned for shifting the firm’s demand curve upward to the level of PM2.
• One point is earned for showing the profit-maximizing quantity, QF2, at MC = new MR.
• One point is earned for stating that ATC at QF2 is lower than PM2.

(d) 1 point:
• One point is earned for stating that the equilibrium quantity will decrease and the equilibrium
price will increase, because the increase in the price of corn causes a decrease in the supply of
cereal.

© 2010 The College Board.


Visit the College Board on the Web: www.collegeboard.com.
AP® MICROECONOMICS
2010 SCORING GUIDELINES

Question 2

5 points (2+2+1)

(a) 2 points:
• One point is earned for the correct side-by-side graphs with a horizontal machine supply curve
for John Lamb (S, D, PR, SM).
• One point is earned for showing the equilibrium rental quantity of machines, QL, at the
intersection of MRP and the horizontal supply curve.

(b) 2 points:
• One point is earned for stating that there will be no change to the marginal product curve for
machine-hours.
• One point is earned for explaining that the MRP curve for machine-hours will decrease (shift to
the left) because the decrease in demand decreases the price of widgets.

(c) 1 point:
• One point is earned for correctly calculating the rental price of a machine:
MPL/w = MPK/r = 28/14 = 60/r. Therefore, r = $30.

© 2010 The College Board.


Visit the College Board on the Web: www.collegeboard.com.
AP® MICROECONOMICS
2010 SCORING GUIDELINES

Question 3

5 points (2+1+2)

(a) 2 points:
• One point is earned for identifying the consumer surplus as P3JM.
• One point is earned for identifying the producer surplus as P1P3M.

(b) 1 point:
• One point is earned for identifying the socially optimal quantity as q1.

(c) 2 points:
• One point is earned for identifying the consumer surplus as P5JK.
• One point is earned for indicating that there is no deadweight loss.

© 2010 The College Board.


Visit the College Board on the Web: www.collegeboard.com.
AP® MICROECONOMICS
2010 SCORING GUIDELINES (Form B)

Question 1

10 points (2 + 2 + 1 + 2 + 2 + 1)

(a) 2 points:
• One point is earned for identifying the output as Q2.
• One point is earned for identifying the price as $7.

(b) 2 points:
• One point is earned for identifying the output as Q4.
• One point is earned for identifying the price as $0.

(c) 1 point:
• One point is earned for stating that the accounting profit is positive, because the firm earns zero
economic profit. (Economic profit = Total revenue - Explicit costs - Implicit costs.)

(d) 2 points:
• One point is earned for identifying the break-even quantity as Q3.
• One point is earned for stating that the demand at Q3 is relatively inelastic.

(e) 2 points:
• One point is earned for stating that the demand curve for bridge crossings will shift down or to the
left.
• One point is earned for stating that the profit-maximizing output will fall.

(f) 1 point:
• One point is earned for stating that building a second bridge would be inefficient and for
explaining that, because there are economies of scale, building a second bridge would raise the
average total cost.

© 2010 The College Board.


Visit the College Board on the Web: www.collegeboard.com.
AP® MICROECONOMICS
2010 SCORING GUIDELINES (Form B)

Question 2

6 points (1 + 2 + 1 + 1 + 1)

(a) 1 point:
• One point is earned for calculating the marginal product of the third worker: $450/$5 per unit =
90 units.

(b) 2 points:
• One point is earned for defining the law of diminishing returns: as more and more units of a
variable input are added to a fixed input, the output increases at a decreasing rate.
• One point is earned for the explanation that diminishing returns occur because of the overuse of
the fixed input.

(c) 1 point:
• One point is earned for stating that diminishing marginal returns first occur with the hiring of the
third worker.

(d) 1 point:
• One point is earned for stating $300.

(e) 1 point:
• One point is earned for stating that the demand for labor will increase because the increase in the
product price raises the marginal revenue product of labor.

© 2010 The College Board.


Visit the College Board on the Web: www.collegeboard.com.
AP® MICROECONOMICS
2010 SCORING GUIDELINES (Form B)

Question 3

5 points (3 + 2)

(a) 3 points:
• One point is earned for stating that the demand for good X is relatively elastic, because the
elasticity coefficient > 1 OR because total revenue rises as price decreases from $30 to $20.
• One point is earned for stating that supply is perfectly inelastic, because there is no change in the
quantity supplied as the price changes OR because the supply elasticity is zero.
• One point is earned for stating that all of the burden of a per-unit tax falls on sellers.

(b) 2 points:
• One point is earned for a correctly labeled graph of supply and demand.
• One point is earned for showing a leftward shift of the demand curve and a decrease in the
equilibrium price.

© 2010 The College Board.


Visit the College Board on the Web: www.collegeboard.com.
AP® MICROECONOMICS
2011 SCORING GUIDELINES

Question 1

10 points (1 + 1 + 1 + 1 + 2 + 2 + 2)

(a) 1 point:
• One point is earned for identifying the profit-maximizing price as $24. ($22 is also acceptable.)

(b) 1 point:
• One point is earned for identifying the profit per unit as $6.

(c) 1 point:
• One point is earned for stating that allocative efficiency is not achieved because price is not equal
to MC or MC is not equal to demand.

(d) 1 point:
• One point is earned for stating that the demand is inelastic because total revenue increases as
price increases from $16 to $18, or because the price elasticity of demand within the price range is
less than 1, or because marginal revenue is negative.

(e) 2 points:
• One point is earned for indicating that the monopolist is not earning positive economic profit,
because price equals average total cost.
• One point is earned for indicating that the monopolist is earning positive accounting profit.

(f) 2 points:
• One point is earned for stating that the marginal revenue of the 8th unit is $22.
• One point is earned for stating that 9 units will be produced.

© 2011 The College Board.


Visit the College Board on the Web: www.collegeboard.org.
AP® MICROECONOMICS
2011 SCORING GUIDELINES

Question 1 (continued)

(g) 2 points:
• One point is earned for stating that 10 units will be produced.
• One point is earned for stating that the consumer surplus is zero.

© 2011 The College Board.


Visit the College Board on the Web: www.collegeboard.org.
AP® MICROECONOMICS
2011 SCORING GUIDELINES

Question 2

6 points (3 + 1 + 2)

(a) 3 points:
• One point is earned for a correctly labeled graph with a horizontal demand curve at the equilibrium
price, PE.
• One point is earned for showing the equilibrium quantity, QE, at MR = MC.
• One point is earned for showing that ATC is below demand or MR at Q.

(b) 1 point:
• One point is earned for showing a leftward shift of the MC curve.

(c) 2 points:
• One point is earned for drawing a correctly labeled graph with a horizontal MFC1 curve at w1 and a
downward-sloping MRP curve and showing QL1.
• One point is earned for shifting the MFC curve up to w2 and showing the new equilibrium quantity
of labor hired, QL2, which is smaller than QL1.

© 2011 The College Board.


Visit the College Board on the Web: www.collegeboard.org.
AP® MICROECONOMICS
2011 SCORING GUIDELINES

Question 3

5 points (4 + 1)

(a) 4 points:
• One point is earned for showing a correctly labeled MSC curve above a correctly labeled MPC
curve.
• One point is earned for drawing the correctly labeled graph with an upward-sloping MPC curve
and a downward-sloping MSB curve and showing the private market equilibrium, Qm.
• One point is earned for showing the allocatively efficient quantity QS with QS < Qm.
• One point is earned for shading the area representing the deadweight loss.

(b) 1 point:
• One point is earned for stating that the deadweight loss does not change because marginal cost
does not change.

© 2011 The College Board.


Visit the College Board on the Web: www.collegeboard.org.
AP® MICROECONOMICS
2011 SCORING GUIDELINES (Form B)

Question 1

10 points (3 + 1 + 2 + 2 + 2)

(a) 3 points:
• One point is earned for a correctly labeled graph of the market showing a downward-sloping
market demand curve, an upward-sloping market supply curve, and Pm and Qm.
• One point is earned for showing the firm’s profit-maximizing quantity, Qf, at MC = MR with a
horizontal demand curve.
• One point is earned for showing the firm’s average total cost (ATC) curve tangent to its demand
curve at Qf.

(b) 1 point:
• One point is earned for stating that Pm is equal to Pf.

(c) 2 points:
• One point is earned for showing a rightward shift of the market demand curve and showing Pm2
and Qm2.
• One point is earned for showing the corresponding upward shift of the firm’s demand curve and
showing Pf2 and Qf2.

(d) 2 points:
• One point is earned for stating that the number of firms increases because the existence of short-
run profits attracts new firms to enter the industry.
• One point is earned for stating that the firm’s ATC curve will shift upward.

(e) 2 points:
• One point is earned for stating that the long-run profit-maximizing price will be higher than Pf.
• One point is earned for stating that the long-run profit-maximizing price will be lower than Pf2.

© 2011 The College Board.


Visit the College Board on the Web: www.collegeboard.org.
AP® MICROECONOMICS
2011 SCORING GUIDELINES (Form B)

Question 2

6 points (3 + 2 + 1)

(a) 3 points:
• One point is earned for a correctly labeled market graph, showing the equilibrium price and
quantity, Pm and Qm.
• One point is earned for showing a MSB curve above a MPB curve and for showing the socially
optimal quantity, Qs.
• One point is earned for shading the area representing the deadweight loss.

(b) 2 points:
• One point is earned for showing an effective (binding) price ceiling below Pm.
• One point is earned for stating that the price ceiling will increase the deadweight loss, because
providers will decrease the quantity of education offered.

(c) 1 point:
• One point is earned for stating that the new equilibrium quantity will be greater than Qm.

© 2011 The College Board.


Visit the College Board on the Web: www.collegeboard.org.
AP® MICROECONOMICS
2011 SCORING GUIDELINES (Form B)

Question 3

5 points (1 + 1 + 3)

(a) 1 point:
• One point is earned for identifying the profit-maximizing quantity of labor as 100 units.

(b) 1 point:
• One point is earned for identifying the wage rate TreeMart pays as $10.

(c) 3 points:
• One point is earned for identifying the quantity of labor in a competitive labor market as 200 units.
• One point is earned for identifying the quantity of labor, 150 units, given a minimum wage of $12.5.
• One point is earned for explaining that the MFC curve (or the supply curve for labor) becomes
horizontal at the minimum wage up to a quantity of 150.

© 2011 The College Board.


Visit the College Board on the Web: www.collegeboard.org.
AP® MICROECONOMICS
2012 SCORING GUIDELINES

Question 1

10 points (5 + 1 + 2 + 2)

(a) 5 points:

(i) 3 points:
• One point is earned for a correctly labeled downward-sloping demand curve and a
downward-sloping marginal revenue curve below the demand curve.
• One point is earned for identifying the profit-maximizing quantity, Qm, at MR = MC.
• One point is earned for identifying price, Pm, above Qm on the demand curve.

(ii) 1 point:
• One point is earned for shading the area of economic loss.

(iii) 1 point:
• One point is earned for identifying the allocatively efficient quantity, Qe, at MC = D.

(b) 1 point:
• One point is earned for stating that the total revenue would decrease because the demand
is price elastic in that range of the demand curve where MR > 0.

(c) 2 points:
• One point is earned for stating that the quantity will increase because the subsidy will
cause the MC curve to shift downward and intersect the MR curve at a larger quantity.
• One point is earned for stating that the consumer surplus will increase.

(d) 2 points:
• One point is earned for stating that the deadweight loss will not change because the lump-
sum subsidy does not change the profit-maximizing quantity.
• One point is earned for stating that economic losses will decrease.

© 2012 The College Board.


Visit the College Board on the Web: www.collegeboard.org.
AP® MICROECONOMICS
2012 SCORING GUIDELINES

Question 2

6 points (3 + 1 + 1 + 1)

(a) 3 points:
• One point is earned for determining the total utility, which is 24.
• One point is earned for stating that three bagels and five toy cars will be purchased.
• One point is earned for explaining that with this combination of bagels and toys, the marginal
utility per dollar spent on bagels equals the marginal utility per dollar spent on toy cars.

(b) 1 point:
• One point is earned for stating that Theresa’s demand for bagels will not change because the
increase in the price of wheat will affect the supply of bagels, not the demand.

(c) 1 point:
• One point is earned for stating that bagels are inferior goods.

(d) 1 point:
• One point is earned for calculating the cross-price elasticity for toy cars and blocks:
-0.04/0.10 = -0.4

© 2012 The College Board.


Visit the College Board on the Web: www.collegeboard.org.
AP® MICROECONOMICS
2012 SCORING GUIDELINES

Question 3

5 points (1 + 3 + 1)

(a) 1 point:
• One point is earned for stating that Loriland is importing 12 million pounds.

(b) 3 points:
• One point is earned for identifying the new level of domestic production as 6 million pounds.
• One point is earned for calculating the domestic consumer surplus as $25 million and showing
the work: ½ [($9 - $4) × 10] = $25
• One point is earned for calculating the revenue from the tariff as $8 million and showing the
work: ($4 - $2) (10 - 6) = $8.

(c) 1 point:
• One point is earned for identifying the per-unit tariff that maximizes the sum of consumer and
producer surplus as $0.

© 2012 The College Board.


Visit the College Board on the Web: www.collegeboard.org.
AP® MICROECONOMICS
2013 SCORING GUIDELINES

Question 1

10 points (4 + 2 + 2 + 1 + 1)

(a) 4 points:
• One point is earned for identifying the quantity, Q1.
• One point is earned for identifying the price, P3.
• One point is earned for identifying the area that represents the profit earned, P1P3ac.
• One point is earned for identifying the area that represents the deadweight loss, acf.

(b) 2 points:
• One point is earned for identifying the quantity produced, Q3.
• One point is earned for identifying the area that represents the total revenue received,
P4fQ30.

(c) 2 points:
• One point is earned for identifying the socially efficient quantity, Q3.
• One point is earned for identifying the area that represents the consumer surplus, P1P4f.

(d) 1 point:
• One point is earned for stating that the monopolist is earning zero economic profit because
the price equals ATC.

(e) 1 point:
• One point is earned for stating that point f is in the inelastic portion of the demand curve
because MR is negative.

© 2013 The College Board.


Visit the College Board on the Web: www.collegeboard.org.
AP® MICROECONOMICS
2013 SCORING GUIDELINES

Question 2

5 points (1 + 2 + 1 + 1)

(a) 1 point:
• One point is earned for stating that PieCrust should choose to advertise because it makes
greater profit by advertising: $250> $180.

(b) 2 points:
• One point is earned for stating that LaPizza does not have a dominant strategy.
• One point is earned for stating that its best choice depends on the strategy chosen by
PieCrust. When PieCrust advertises, LaPizza does better by not advertising ($300>$200);
When PieCrust does not advertise, LaPizza does better by advertising ($500 >$400).

(c) 1 point:
• One point is earned for identifying the daily profit for PieCrust, $450, and for LaPizza, $300.

(d) 1 point:
• One point is earned for reproducing the payoff matrix to reflect the effect of the increase in
advertising costs.

LaPizza
Advertise Not Advertise

Advertise $190, $140 $390, $300


PieCrust

Not Advertise $180, $440 $390, $400

© 2013 The College Board.


Visit the College Board on the Web: www.collegeboard.org.
AP® MICROECONOMICS
2013 SCORING GUIDELINES

Question 3

6 points (1 + 3 + 2)

(a) 1 point:
• One point is earned for drawing a correctly labeled supply and demand graph with PE and QE
labeled.

(b) 3 points
• One point is earned for drawing the MSC curve above the supply curve.
• One point is earned for labeling the demand curve MSB.
• One point is earned for shading the correct area of the deadweight loss.

(c) 2 points:
• One point is earned for stating that the market equilibrium quantity is less than the socially
optimal quantity because the fireworks generate a positive externality or provide an external
benefit to third parties.
• One point is earned for stating that the deadweight loss will increase.

© 2013 The College Board.


Visit the College Board on the Web: www.collegeboard.org.







o
o
o
o

You might also like